67
Exercices sur les intégrales généralisées 1. Convergence et calcul d’intégrales généralisées. 2. Intégrales généralisées et séries. 3. Intégrales généralisées et sommes de Riemann. 4. Intégration des relations de comparaison. 5. Intégrales semi-convergentes. 6. Espaces fonctionnels. 7. Convergence monotone, convergence dominée 8. Intégration terme à terme des séries. 9. Intégrales dépendant d’un paramètre. 10. Intégrales eulériennes. 11. Transformation de Laplace. Pierre-Jean Hormière ____________ « Je mets beaucoup d’ordre dans mes idées. Ça ne va pas tout seul. Il y a des idées qui ne supportent pas l’ordre et qui préfèrent crever. À la fin, j’ai beaucoup d’ordre et presque plus d’idées. » Norge Cher Norge, ce n’est pas bien du tout de vous moquer de ceux qui mettent de l’ordre dans leurs idées ! Ne seriez-vous pas un peu… belge sur les bords ? Pour ceux qui l’ignorent, la Belgique est cet improbable petit pays qui est resté plus d’un an sans gouvernement tout en réalisant 2,7 % de croissance, ceci expliquant peut-être cela… Et de surcroît, en sus d’être belge, ne seriez-vous pas un peu poète ? Poète belge ! Ah ! vous ne tiendriez pas de tels discours si Napoléon avait gagné la bataille de Waterloo : la Belgique serait un département français, et vous auriez fait de solides études mathématiques dans nos glorieuses classes préparatoires. On y apprend à classer ses idées, afin de diriger le monde, et non à finir antiquaire à Saint-Paul de Vence ! 1. Convergence et calcul d’intégrales généralisées . Exercice 1 : Soient I un intervalle de R, f, g, h trois fonctions réglées de I dans R telle que f g h. Montrer que si f et h sont intégrables, il en est de même de g. Solution : Nous avons f g h , donc - h -g - f . Par conséquent, | g | = max(g, -g ) max( h, -f ). Or max( h, - f ) = 2 1 [ h f + | h + f | ] est intégrable. Exercice 2 : Convergence et calcul de I(a, b) = + + + 0 ) )( ( b x a x dx ( a et b > 0 ). Continuité de I ? Solution :

Exercices intégrales généralisées

  • Upload
    others

  • View
    8

  • Download
    0

Embed Size (px)

Citation preview

Page 1: Exercices intégrales généralisées

Exercices sur les intégrales généralisées

1. Convergence et calcul d’intégrales généralisées.

2. Intégrales généralisées et séries.

3. Intégrales généralisées et sommes de Riemann.

4. Intégration des relations de comparaison.

5. Intégrales semi-convergentes.

6. Espaces fonctionnels.

7. Convergence monotone, convergence dominée

8. Intégration terme à terme des séries.

9. Intégrales dépendant d’un paramètre.

10. Intégrales eulériennes.

11. Transformation de Laplace.

Pierre-Jean Hormière ____________

« Je mets beaucoup d’ordre dans mes idées. Ça ne va pas tout seul. Il y a des idées qui ne supportent pas l’ordre et qui préfèrent crever. À la fin, j’ai beaucoup d’ordre et presque plus d’idées. »

Norge

Cher Norge, ce n’est pas bien du tout de vous moquer de ceux qui mettent de l’ordre dans leurs idées ! Ne seriez-vous pas un peu… belge sur les bords ? Pour ceux qui l’ignorent, la Belgique est cet improbable petit pays qui est resté plus d’un an sans gouvernement tout en réalisant 2,7 % de croissance, ceci expliquant peut-être cela… Et de surcroît, en sus d’être belge, ne seriez-vous pas un peu poète ? Poète belge ! Ah ! vous ne tiendriez pas de tels discours si Napoléon avait gagné la bataille de Waterloo : la Belgique serait un département français, et vous auriez fait de solides études mathématiques dans nos glorieuses classes préparatoires. On y apprend à classer ses idées, afin de diriger le monde, et non à finir antiquaire à Saint-Paul de Vence !

1. Convergence et calcul d’intégrales généralisées.

Exercice 1 : Soient I un intervalle de R, f, g, h trois fonctions réglées de I dans R telle que f ≤ g ≤ h. Montrer que si f et h sont intégrables, il en est de même de g.

Solution : Nous avons f ≤ g ≤ h , donc − h ≤ −g ≤ − f . Par conséquent, | g | = max(g, −g ) ≤ max( h, −f ).

Or max( h, − f ) = 21 [ h – f + | h + f | ] est intégrable.

Exercice 2 : Convergence et calcul de I(a, b) = ∫+∞

++0 ))(( bxaxdx ( a et b > 0 ). Continuité de I ?

Solution :

Page 2: Exercices intégrales généralisées

2

La fonction f(x) = ))((

1bxax ++ est continue positive sur R+, et O(

²1x

) au V(+∞), ou ≤ ²

1x

, donc

intégrable. Pour calculer I(a, b), décomposons la fraction en éléments simples.

On obtient, si a ≠ b : ))((

1bxax ++ =

ab−1 (

ax+1 −

bx+1 ) (*).

Attention ! Ne pas écrire I(a, b) = ab−

1 ( ∫+∞

+0 axdx − ∫

+∞

+0 bxdx ), mais

I(a, b) = limA→+∞ ab−

1 ( ∫ +A

axdx

0 − ∫ +

A

bxdx

0) = limA→+∞

ab−1 (ln(A + a) − ln a − ln(A + b) + ln b )

= limA→+∞ ab−

1 ( lnbAaA

++ − ln

ba ) =

abab

−−lnln .

Pour calculer la limite, il y a intérêt à solidifier les logarithmes en un seul bloc. Comme la somme

des résidus est nulle, la limite en l’infini est nulle. Si a = b, on trouve, I(a, a) = a1 .

Conclusion : Pour a et b > 0, I(a, b) = ab

ab−−lnln si a ≠ b , I(a, a) =

a1 .

Montrons que la fonction (a, b) → I(a, b) est continue sur R*+×R*+. Cela vient de ce que les hypothèses (H1), (H2) et (H3) du théorème de continuité des intégrales à paramètres s’appliquent. En particulier (H3) :

Pour a ≥ α > 0 et b ≥ β > 0, 0 ≤ ))((

1bxax ++ ≤

))((1

βα ++ xx, majorante intégrable…

Exercice 3 : Convergence et calcul de I = ∫+∞

+++0 )3)(2)(1( xxxdx et J = ∫

+∞

+++0 )²3)²(2)²(1( xxxdx .

Solution :

Chacune des fonctions intégrées f et g est continue > 0 et O(1/x2) au V(+∞), donc intégrable.

Pour calculer I et J, décomposons f et g en éléments simples.

Décomposons )3)(2)(1(

1+++ xxx

en éléments simples.

On obtient : )3)(2)(1(

1+++ xxx

= 21

11+x

− 2

1+x

+ 21

31+x

(*).

Attention ! Ne pas écrire I = 21 ∫

+∞

+0 1xdx − ∫

+∞

+0 2xdx +

21 ∫

+∞

+0 3xdx , mais

I = limA→+∞ 21 ∫ +

A

xdx

0 1 − ∫ +

A

xdx

0 2 +

21 ∫ +

A

xdx

0 3

= limA→+∞ 21 ln(A + 1) − ln(A + 2) +

21 ln(A + 3) + ln 2 −

21 ln 3

= limA→+∞ ln2

)3)(1(+

++A

AA + ln 2 −

21 ln 3 = ln 2 −

21 ln 3

Pour calculer la limite, il y a intérêt à solidifier les logarithmes en un seul bloc. Comme la somme des résidus est nulle, la limite en l’infini est nulle.

Pour calculer J, élévons (*) au carré. Il vient : )²3)²(2)²(1(

1+++ xxx

= 41

)²1(1+x

+ )²2(

1+x

+ 41

)²3(1+x

− )2)(1(

1++ xx

− )3)(2(

1++ xx

+ 21

)3)(1(1

++ xx .

Il reste à intégrer chaque terme…

La situation est analogue au calcul de ∑+∞

= +++1 )3)(2)(1(1

n nnn et ∑

+∞

= +++1 )²3)²(2)²(1(1

n nnn.

> f:=1/((x+1)*(x+2)*(x+3));convert(f,parfrac,x);

Page 3: Exercices intégrales généralisées

3

int(f,x);int(f,x=0..infinity);

> g:=f^2;convert(g,parfrac,x);int(g,x);int(g,x=0..infinity);

Exercice 4 : Nature, calcul des intégrales ∫+∞

+++0 ))...(2).(1( nxxxdx et ∫

+∞

+++0 )²)²...(2)².(1( nxxxdx .

Solution : Notons In et Jn ces intégrales respectives, Fn et Gn les fonctions intégrées.

Il est clair que In existe pour n ≥ 2, Jn pour n ≥ 1.

Fn(x) = ∑= +n

k

k

kxA

1

, où Ak = !)1( 1

n

k−−k knC ; on a ∑

=

n

kkA

1

= 0 ( car x.Fn(x) → 0 quand x → +∞ ).

En solifidifiant les logarithmes comme ci-dessus, il vient : In = !

1n ∑=

−n

k

knk kkC1

ln..)1( .

Gn(x) = ∑= +n

k

k

kxB

1

( + )²( kX

Ck

+ ) , il vient aussitôt Jn = ∑=

n

k

k

kC

1

( − Bk ln k )

Or Ck = )²!(²

nk ( knC )

2 par les techniques habituelles, mais il n’est pas facile d’obtenir Bk .

Mieux vaut noter que Gn(x) = Fn(x)2 = ∑

= +n

k

k

kxA

1 )²()²(

+ 2∑< ++qp

qp

qXpXAA

))((. …

Remarque : On peut obtenir des équivalents des suites (In) et (Jn) à l’aide de la méthode de Laplace.

Exercice 5 : Convergence et calcul de I = ∫+∞

∞− +14xdx et J = ∫

+∞

∞− + dxxx .

4 .

Solution : L’intégrabilité des fonctions continues positives f(x) = 1

14+x

et g(x) = 1

²4+xx ne pose

aucun problème : elles sont toutes deux O(1/x²) au V(±∞). 1ère méthode : on peut les calculer séparément par calcul des primitives. > f:=1/(x^4+1);g:=x^2/(x^4+1); convert(f,parfrac,x,sqrt(2));convert(g,parfrac,x,sqrt(2));

> Int(1/(x^4+1),x)=int(1/(x^4+1),x);Int(x^2/(x^4+1),x)=int(x^2/(x^4+1),x);

− + 14

− + 2 x 2

− + x2 x 2 1

14

( ) + 2 x 2

+ + x2 x 2 1 := f

1

+ x4 1

:= gx2

+ x4 1 −

14

x 2

− + x2 x 2 1

14

x 2

+ + x2 x 2 1

:= f1

( ) + x 1 ( ) + x 2 ( ) + x 3 − + 12

1 + x 1

1 + x 2

12

+ x 3

− + 12

( )ln + x 1 ( )ln + x 212

( )ln + x 3 − + 12

( )ln 3 ( )ln 2

:= g1

( ) + x 1 2 ( ) + x 2 2 ( ) + x 3 2 − + + + 14

1

( ) + x 1 2

34

1 + x 1

1

( ) + x 2 2

14

( ) + x 3 2

34

+ x 3

− − − − + 14

1 + x 1

34

( )ln + x 11

+ x 214

1 + x 3

34

( )ln + x 3 − + 34

( )ln 356

Page 4: Exercices intégrales généralisées

4

= d⌠

1

+ x4 1x + +

18

2

ln

+ + x2 x 2 1

− + x2 x 2 1

14

2 ( )arctan + x 2 114

2 ( )arctan − x 2 1

= d⌠

x2

+ x4 1x + +

18

2

ln

− + x2 x 2 1

+ + x2 x 2 1

14

2 ( )arctan + x 2 114

2 ( )arctan − x 2 1

> Int(1/(x^4+1),x=-infinity..infinity)=int(1/(x^4+1), x=-infinity..infinity);Int(x^2/(x^4+1),x=-infinity..infinity) =int(x^2/(x^4+1),x=-infinity..infinity);

= d⌠

−∞

1

+ x4 1x

12

π 2

2ème méthode : on peut les calculer simultanément. Tout d’abord, elles sont égales, car le changement de variable y = 1/x donne :

I = ∫+∞

∞− +14xdx = 2∫

+∞

+0 4 1xdx = 2∫

+∞

+0 4 1².

ydyy

= ∫+∞

∞− +1².4ydxy

= J.

Calculons I + J au moyen du changement de variable u = x – 1/x :

I + J = dxxx .

11²

4∫+∞

∞− ++ = 2 dx

xx .

11²

0 4∫+∞

++ = 2 dx

xx

x .

²1²

11

0 2∫∞+

+

+ = 2∫

+∞

+0 2 2udu = 2 Arctan

2u +∞

0 = π 2 .

Par conséquent I = J = 22π .

Remarque : Arnaudiès suggère le changement de variable x = et dans I et J.

3ème méthode : intégration complexe. (hors programme)

Soit γ le lacet obtenu en parcourant le segment [− R, R] et le demi-cercle de centre O et de rayon R situé dans le demi-plan Re z > 0, parcouru dans le sens trigonométrique (R > 1).

Calculons de deux façons A(R) =∫ +γ 14zdz .

D’une part, A(R) = ∫+

− +R

R xdx

14 + ∫ +π

θ

θ θ0 44 1

Rei

i

eRdi

→ ∫+∞

∞− +14xdx quand R → +∞

car |∫ +π

θ

θ θ0 44 1

Rei

i

eRdi

| ≤ ∫ −π θ0 4 1R

Rd = 14−R

Rπ → 0 quand R → +∞.

D’autre part, si α = exp(iπ/4), 1

14+z

= 41− ( α

α−z

+ ααiz

i− + α

α+−z

+ ααizi

+− ) .

donc : A(R) = 41− 2iπ ( α.I(γ, α) + iα.I(γ, iα) − α.I(γ, −α) − iα.I(γ, −i.α) ) .

Ces indices valant respectivement 1, 1, 0 et 0, A(R) = 41− 2iπ α ( 1 + i ) =

22π .

Exercice 6 : Montrer que ∫+∞

+0.

²1ln dx

xx converge et vaut 0.

Solution : 1) Convergence.

La fonction f(x) = ²1

lnxx

+ est continue sur ]0, +∞[, négative sur ]0, 1], positive sur [1, +∞[.

Au V(0+), f(x) ∼ ln x, qui est intégrable.

Au V(+∞), f(x) ∼ ²

lnxx = O( 2/3

1x

), donc f est également intégrable.

= d⌠

−∞

x2

+ x4 1x

12

π 2

Page 5: Exercices intégrales généralisées

5

2) Calcul. Bien que f ne se primitive pas élémentairement, on peut calculer I = ∫+∞

+0.

²1ln dx

xx .

Le changement de variable y = 1/x donne I = − I, donc I = 0.

Plus précisément, le même changement de variable donne ∫ +ε

ε

/1.

²1ln dx

xx = 0 pour tout ε > 0.

3) Maple donne ceci, qui intrigue : > int(ln(x)/(1+x^2),x);

− + − + 12

I ( )ln x ( )ln + 1 I x12

I ( )ln x ( )ln − 1 I x12

I ( )dilog + 1 I x12

I ( )dilog − 1 I x

> int(ln(x)/(1+x^2),x=0..infinity); 0

> int(ln(t)/(1+t^2),t=1..x); 12

I ( )ln x ( )ln + 1 I x12

I ( )ln x ( )ln − 1 I x12

I ( )dilog + 1 I x12

I ( )dilog − 1 I x− + − +

Catalan +

Exercice 7 : Nature et calcul de l’intégrale ∫+

3

2.ln dxx .

Nature de l’intégrale ∫+

3

2 xdx ? Trouver lim ε→0+ ∫

ε

1 xdx + ∫

+

+

3

ε xdx . Explications ?

Solution : La première intégrale est impropre en 0, mais converge, car ln |x| est intégrable au V(0).

∫+

3

2.ln dxx = ∫

3

0.ln dxx + ∫

2

0.ln dxx = 3.ln 3 – 3 + 2.ln 2 – 2 = ln(4.27) – 5.

La fonction 1/x n’est intégrable ni sur ]0, 3], ni sur [−2, 0], par conséquent la seconde intégrale diverge. Cependant, elle converge en un sens affaibli :

∫−

ε

2 xdx + ∫

+

+

3

ε xdx = ln(ε) − ln 2 + ln 3 − ln(ε) = ln 3 – ln 2.

Variante : par imparité,

∫−

ε

2 xdx + ∫

+

+

3

ε xdx = ∫

ε

2 xdx + ∫

+

+

2

ε xdx + ∫

+

+

3

2 xdx = ∫

+

+

3

2 xdx = ln 3 – ln 2.

On dit que ∫+

3

2 xdx converge en valeur principale de Cauchy, et l’on note v.p.∫

+

3

2 xdx = ln 3 – ln 2.

Exercice complémentaire : calculer v.p. ∫+∞

−−−0 )3)(2)(1( xxxdx .

Exercice 8 : Convergence et calcul des intégrales ∫+

− −1

1 ²1 xdx et ∫ −−

b

a attbdt

))(( (a < b)

Solution :

La fonction x → ²1

1x−

est continue positive et non bornée sur ]−1, 1[. L’intégrale est donc

impropre en +1 et −1. Mais on dispose d’une primitive élémentaire :

∫ −d

c xdx

²1 = Arcsin d – Arcsin c, donc à la limite∫

+

− −1

1 ²1 xdx = Arcsin 1 – Arcsin –1 = π.

Le changement de variable t = 2ba+ + uab .

2− ramène aussitôt la seconde intégrale à la première :

Page 6: Exercices intégrales généralisées

6

∫ −−b

a attbdt

))(( = ∫

+

− −1

1 ²1 udu = π.

Exercice 9 : Existence et calcul des intégrales ( n ∈ N ) :

In = ∫+∞

−0

.. dtet tn , Jn = ∫ −1

0.)ln( duu n , Kn = ∫

+∞

∞− + nxdx

)1²( , Ln = ∫

+∞

∞− tchdt

n .

Solution : 1) Cet exercice peut être traité de deux façons légèrement différentes.

1ère approche. Posons, pour tout x, Fn(x) = ∫ −x

tn dtet0

.. .

F0(x) = 1 − e−x

→ 1 quand x → +∞, donc I0 converge et vaut 1.

Une intégration par parties donne Fn(x) = − xn

e−x

+ n Fn−1(x). Or xn

e−x

→ 0 en +∞.

Si Fn−1(x) a une limite In−1 en +∞, Fn(x) a aussi une limite, et cette limite vaut In = n.In−1.

Ainsi, toutes les intégrales convergent par récurrence, et I0 = 1 et In = n.In−1 impliquent In = n!.

2ème approche. Montrons d’abord que toutes ces intégrales convergent.

En effet, 0 ≤ tn

e−t

≤ 1/t2 pour t assez grand, car t

n+2 e

−t → 0 en +∞.

Ou bien 0 ≤ tn

e−t

≤ e−t/2

pour t assez grand, car tn

e−t/2

→ 0 en +∞.

Or les fonctions tests 1/t2 et e

−t/2 sont intégrables sur [1, +∞[.

Dès lors, une ipp donne, pour n > 0 : In = [− tn

e−t

] +∞0 + n.In−1 = n.In−1 .

Comme I0 = 1, on conclut par récurrence que In = n !.

2) On peut étudier des intégrales directement : existence et IPP.

On peut aussi noter que le chgt de variable t = ln u donne Jn = ∫ −1

0.)ln( duu n = ∫

+∞−

0.. dtet tn = n!

3) Kn = ∫+∞

∞− + nxdx

)1²( est l’intégrale impropre en ±∞ de nx )1²(

1+ , fonction continue positive.

Par parité, il suffit de se placer en +∞. Or 0 ≤ nx )1²(1+ ∼ nx2

1 qui est intégrable ssi n > 0.

Il est clair que K1 = π. Une intégration par parties donne 2n Kn+1 = (2n – 1)Kn .

Donc Kn = 2232

−−

nn

4252

−−

nn …

21 K1 =

)!²1(2)!22(

22 −−

− nn

n π.

Remarque : Les Kn sont des intégrales de Wallis, car le changement de variable x = tan θ, ou plutôt

θ = Arctan x, donne Kn = ∫+

−−

2/

2/

22 .cosπ

πθθ dn = 2W2n−2.

4) Ln = ∫+∞

∞− tchdt

n est l’intégrale impropre en ±∞ de tchn

1 , fonction continue positive.

Par parité, il suffit de se placer en +∞. Or 0 ≤ tchn

1 ∼ nt

n

e2

, fonction intégrable.

Ln existe pour n ≥ 1. Une intégration par parties Ln = ∫+∞

∞− + tchdtcht

n 1. = ∫

+∞

∞− + tchshtdn 1

)( = …

donne (n + 1).Ln+2 = (n + 2).Ln . Tout revient à calculer L1 et L2.

Remarques : i) La situation est analogue à celle des intégrales de Wallis.

D’ailleurs Ln est une intégrale de Wallis, si l’on passe par la transformation de Gudermann.

Page 7: Exercices intégrales généralisées

7

En effet le changement de variable y = Arctan (sh x) donne Ln = ∫+

−−

2/

2/

1 .cosπ

πdyyn = 2Wn−1.

ii) La suite (Ln) tend en décroissant vers 0 par convergence dominée.

La méthode de Laplace fournit, quant à elle, un équivalent de Ln :

Ln = ∫+∞

∞−− dte chtn .)ln(. ∼ ∫

+∞

∞−− dte nt .2/² =

nπ2 .

Exercice 10 : Convergence et calcul de ∫−1

0.

²²)1ln( dt

tt

.

Solution : 1) C’est l’intégrale impropre en 0 et 1 de la fonction f(t) = ²

²)1ln(t

t− continue et < 0.

En 0+, f(t) → − 1 : l’intégrale est faussement impropre.

En 1−, f(t) ∼ ln(1 − t2) ∼ ln(1 − t), qui est intégrable.

2) Intégrons par parties : ∫− dt

tt .

²²)1ln(

= −t

t²)1ln( − + ln |

11

+−

tt | .

La limite de cette primitive en 0 étant nulle ( f est d’ailleurs C1 sur [0, 1[ ) , intégrons sur [0, 1 − ε] :

∫− −ε1

0.

²²)1ln( dt

tt

= − εε

−−−

1)²)1(1ln(

+ ln | εε

−−2

| = − εε

−1ln − ε

ε−

−1

)2ln( + ln ε − ln(2 − ε) → −2 ln 2 ,

car − εε

−1ln + ln ε = − ε

εε−1ln. → 0 quand ε → 0.

3) Autre approche, fort différente :

Développons f en série entière sur [0, 1[ : f(t) = − 1 − 2²t −

3

4t −

4

6t − …

En intégrant terme à terme cette série, il vient formellement : ∫−1

0.

²²)1ln( dt

tt

= − ∑+∞

= −1 )12(1

n nn .

Or ∑= −N

n nn1 )12(1 = ∑

=−−

N

n nn1

)112

2( = 2 ( H2N – 21 HN ) − HN = 2 H2N – 2 HN → 2 ln 2.

L’intégration terme à terme est justifiée, soit par associativité de bornes supérieures, soit par appli-

cation du théorème ∑∫ nu < +∞.

Exercice 11 : Nature et calcul des intégrales : ∫ ++−1

0 11 xxdx , ∫ ++−

1

0 ²1²1 xxdx .

Solution : [ Mines 1982, X 1992, Centrale PC 2010, etc. ]

I = ∫ ++−1

0 11 xxdx n’est pas une intégrale impropre. Pour la calculer, deux méthodes :

1ère méthode : poser x = cos(2θ), ou plutôt 2θ = Arccos x . Il vient : I = 2 ∫ +4/

0 cossin).2sin(π

θθθθ d

.

Les règles de Bioche ne s’appliquent pas, mais on peut exploiter la symétrie sin-cos en notant que :

I = ∫ +4/

0 )4/sin().2sin(π

πθθθ d

= ∫−2/

4/.

sin)2cos(π

πϕϕ

ϕ d = ∫−2/

4/.

sin1²sin2π

πϕϕ

ϕ d = 2 − ∫2/

4/ sinπ

π ϕϕd

.

= 2 − ln( tan2ϕ

) 2/4/

ππ = 2 + ln tan

8π = 2 + ln( 2 − 1 ).

2ème méthode : Ecrivons I = dxx

xx .2

111

0∫−−+ = dx

xx.

211

0∫+ − dx

xx.

211

0∫− …

Mais attention, cette idée est fausse, car les deux intégrales divergent.

Page 8: Exercices intégrales généralisées

8

Ecrivons donc I = limα→0+ dxx

xx .2

111

∫ −−+α

= limα→0+ dxxx.

211

∫ +α

− dxxx.

211

∫ −α

.

Calculons séparément ces deux intégrales en posant y = x±1 :

dxxx.

211

∫ +α

= dyyy .

1²²2

1∫ + −α = y +

21 ln|

11

+−

yy | 2

1 α+ ,

dxxx.

211

∫ −α

= dyyy .

1²²0

1∫ − −α = y +

21 ln|

11

+−

yy | 0

1 α− .

dxx

xx .2

111

∫ −−+α

= 2 + 21 ln

1212

+− − α−1 −

21 ln

1111

++−+

αα + α−1 +

21 ln |

1111

+−−−

αα |

→ 2 + 21 ln

1212

+− , car ln(

1111

−+++

αα

1111

+−−−

αα ) → 0 quand α → 0.

3ème méthode : calculer I = dxx

xx .2

111

0∫−−+ par développement en série entière.

Avec Maple : > Int(1/(sqrt(1+x)+sqrt(1-x)),x=0..1)=int(1/(sqrt(1+x)+sqrt(1-x)),x=0..1); evalf(%);

= d⌠

0

1

1

+ + 1 x − 1 xx + − 2

12

( )ln − 2 112

( )ln + 1 2

= .5328399754 .5328399746 > Int(sqrt(2)*sin(2*t)/(sin(t)+cos(t)),t=Pi/4..Pi/2) =int(sqrt(2)*sin(2*t)/(sin(t)+cos(t)),t=Pi/4..Pi/2); solve(2*t/(1-t^2)=1,t);simplify(convert(simplify(subs(tan(1/8*Pi)=sqrt(2)-1,2*(-sqrt(2)*tan(1/8*Pi)+arctanh(1/2*(tan(1/8*Pi)-1)*sqrt(2)) *tan(1/8*Pi)^2+sqrt(2)+arctanh(1/2*(tan(1/8*Pi)-1)*sqrt(2))) /(1+tan(1/8*Pi)^2))),ln));evalf(%);

d⌠

/1 4π

/1 2 π

2 ( )sin 2t + ( )sin t ( )cos t

t 2 2

tan

18

π

arctanh

12

tan

18

π 1 2

tan

18

π2

− =

2

arctanh

12

tan

18

π 1 2 − − + 1

tan

18

π2

,− − 1 2 − 2 1

12

− − + + − 4 4 2 4 ( )ln − + 2 2 2 ( )ln 2 2 2 ( )ln − + 2 2 2 ( )ln 2

− 2 2

.5328399745 > Int(sqrt(1+x)/(2*x)-sqrt(1-x)/(2*x),x=0..1)=int(sqrt(1+x)/(2*x)-sqrt(1-x) /(2*x),x=0..1);

= d⌠

0

1

− 12

+ 1 xx

12

− 1 xx

x + − 212

( )ln − 2 112

( )ln + 1 2

> Int(sqrt(1+x)/(2*x),x=0..1)-Int(sqrt(1-x)/(2*x),x=0..1) =int(sqrt(1+x)/(2*x),x=0..1)-int(sqrt(1-x)/(2*x),x=0..1);

= − d⌠

0

1

12

+ 1 xx

x d⌠

0

1

12

− 1 xx

x undefined

> Int(sqrt(1+x)/(2*x),x=epsilon..1)-Int(sqrt(1-x)/(2*x),x=epsilon..1) =int(sqrt(1+x)/(2*x),x=epsilon..1)-int(sqrt(1-x)/(2*x),x=epsilon..1);

Page 9: Exercices intégrales généralisées

9

− d⌠

ε

1

12

+ 1 xx

x d⌠

ε

1

12

− 1 xx

x 212

( )ln − 2 112

( )ln + 1 2 + 1 ε + − − =

12

( )ln − + 1 ε 112

( )ln + 1 + 1 ε12

I π − 1 ε12

( )ln − − 1 ε 1 − + − + +

12

( )ln + 1 − 1 ε −

> f:=epsilon->sqrt(2)+1/2*ln(sqrt(2)-1)-1/2*ln(1+sqrt(2))-sqrt(1+epsilon)-1/2*ln(sqrt(1+epsilon)-1)+1/2*ln(1+sqrt(1+epsilon))+sqrt(1-epsilon) +1/2*ln(1-sqrt(1-epsilon))-1/2*ln(1+sqrt(1-epsilon)); series(f(epsilon),epsilon=0,2);

f ε 212

( )ln − 2 112

( )ln + 1 2 + ε 112

( )ln − + ε 1 1 + − − − → :=

12

( )ln + 1 + ε 1 − 1 ε12

( )ln − 1 − 1 ε12

( )ln + 1 − 1 ε + + + −

− +

+ − 2

12

( )ln − 2 112

( )ln + 1 212

ε ( )O ε2

J = ∫ ++−1

0 ²1²1 xxdx , également non impropre, se traite de la même façon.

∫ ++− ²1²1 xxdx = dx

xxx .

²2²1²1∫ −−+ = dx

xx .²2²1∫ + − dx

xx .²2²1∫ −

Ces intégrales se calculent par intégrations par parties ou par changement de variable

dxxx .²2²1∫ + = − ²1

21 xx

+ + 21 Argsh x , dx

xx .²2²1∫ − = − ²1

21 xx

− − 21 Arcsin x .

Ecrivons I = limα→0+ ∫ ++−1

²1²1α xxdx = limα→0+ dx

xx .²2²11

∫ +α

− dxxx .²2²11

∫ −α

.

Au final, on trouve J = 4π −

22 +

21 ln(1 + 2 ).

> int(1/(sqrt(1+x^2)+sqrt(1-x^2)),x=0..1);

− − + 12

212

( )ln − 2 114

π

> f:=int(sqrt(1+x^2)/(2*x^2),x)-int(sqrt(1-x^2)/(2*x^2),x);

f12

( ) − 1 x2( )/3 2

x12

x − 1 x2 12

( )arcsinx12

( ) + 1 x2( )/3 2

x12

x + 1 x2 + + − + :=

12

( )arcsinhx +

> subs(x=1,f)-limit(f,x=0);

− + 12

( )arcsin 112

212

( )arcsinh 1

Exercice 12 : Convergence et calcul de I = ∫2/

0.tan

πdxx et J = ∫

2/

0).ln(tan.cos

πdxxx .

Solution : • I est l’intégrale impropre en 2π de la fonction f(x) = xtan qui est continue positive et

équivalente à xcos

1 = )

2sin(

1

x−π ∼

x−2

au V(2π ).

Les changements de variable t = tan x, puis u = t donnent :

Page 10: Exercices intégrales généralisées

10

I = dtt

t .1²0∫

+∞

+ = duu

u .1²2

0 4∫+∞

+ = 22π si l’on en croit l’exercice 3.

• J est l’intégrale imprope en 0 et 2π de la fonction f(x) = cos x.ln(tan x) qui est continue, positive

sur [4π ,

2π [, négative sur ]0,

4π ].

Au V(0+), f(x) ∼ ln(tan x) ∼ ln x, car ln(tan x) = ln(x + O(x3)) = ln x + ln(1 + O(x

3)) = ln x + O(x

2).

Or ln x est intégrable sur ]0, 1].

Au V(2π −), x =

2π − h, donc f(x) = − sin h.ln(tan h) ∼ − h.ln h ; il y a fausse impropreté.

Pour calculer J = ∫2/

0).ln(tan.cos

πdxxx , intégrons par parties.

Mais pour éviter tout problème, cherchons une primitive de f par parties.

∫ dxxx ).ln(tan.cos = sin x.ln(tan x) − ln | tan(2x +

4π ) | = F(x).

lim x→0+ F(x) = 0 car sin x.ln(tan x) → 0 en 0+ (voir ci-dessus).

En 2π −, x =

2π − h, et F(x) = − cos h.ln(tan h) + ln(tan

2h )

= cos h.ln(cos h) − cos h.ln(sin h) + ln (sin2h ) − ln (cos

2h ) = − cos h.ln(sin h) + ln (sin

2h ) + o(1).

Or − cos h.ln(sin h) + ln (sin2h ) = − cos h.( ln h + O(h)) + ln

2h + O(h) → − ln 2.

Conclusion : ∫2/

0.tan

πdxx =

22π , ∫

2/

0).ln(tan.cos

πdxxx = − ln 2.

Variante : le changement de variable t = tan x donne I = ∫+∞

+0 2/3 .²)1(

ln dttt .

Exercice 13 : Nature et calcul des intégrales suivantes :

∫∞+

+0.

)²1²(dx

xx , ∫

+∞

+++0 1²)1( xxxdx , ∫

+∞

++0 1²1 xxdx , dx

xxx

.)1(

ln.0 34

3

∫∞+

+ , ∫+∞

+0 3/2).1( xxdx ,

dx

xxx

xx

.)

11(

11

0∫−+

− , ∫ −+1

0 )1()2( xxxdx .

Solution :

1) Le chgt de variable t = x donne ∫∞+

+0.

)²1²(dx

xx = ∫

+∞

+0 4 )²1(².2

tdtt .

Nous voilà ramenés à une fraction rationnelle... Décomposer en éléments simples, passer par la variable complexe, ou faire appel à Maple…

2) f(x) = 1²)1(

1+++ xxx

est continue positive sur R+, équivalente à 21x

au V(+∞), donc intégrable.

On a x2 + x + 1 = ( x +

21 )2

+ 43 : poser x = −

21 +

23 sh t, etc.

3) f(x) = 1²1

1++ xx

est continue positive sur R+, équivalente à 21x

au V(+∞), donc intégrable.

Le changement de variable x = sh t donne ∫+∞

++0 1²1 xxdx = ∫

+∞

+0 .1.chtshtdtcht .

Fraction rationnelle en ch-sh…

Page 11: Exercices intégrales généralisées

11

4) f(x) = 34

3

)1(ln.+x

xx est continue, tend vers 0 en 0+, et est O(8

1x

) au V(+∞). Elle est donc intégrable.

Le changement de variable u = x4 donne dx

xxx

.)1(

ln.0 34

3

∫∞+

+ = 161 du

uu .)1(

ln0 3∫+∞

+ .

Intégrons par parties : duu

u .)1(

ln3∫ + =

)²1(2ln+

−u

u + 21 ∫ + )²1(uu

du = )²1(2

ln+

−u

u + 21 ln

1+uu +

21

11+u

.

Limite en +∞ : 0, limite en 0+ : 21 . Au final : dx

xxx

.)1(

ln.0 34

3

∫∞+

+ = −321 .

f(x) = 3/2)1(1

xx+ est continue positive sur ]0, +∞[, intégrable car f(x) ∼ 3/21

x en 0+, ∼ 3/4

1x

en +∞.

5) Le changement de variable u = x1/3

donne : ∫+∞

+0 3/2).1( xxdx = ∫

+∞

+0 3 .1

3 duu

.

1

33+u

= 1

1+u

+ 1²

2+−

+−uu

u = 1

1+u

− 21

1²12+−

−uu

u + 23

1²1

+−uu.

Donc : ∫ + duu

.1

33 = ln

1

+−+uu

u + 3 Arctan

312 −u . Au final :

∫+∞

+0 3/2).1( xxdx = ∫

+∞

+0 3 .1

3 duu

= 3

32π .

6) f(x) = )

11(

1

xxx

xx

−+− est continue positive sur ]0, 1[, f(x) ∼

x1 en 0+, à 1 en 1 (fausse impropreté).

Les changements de variable x = sin2 θ , θ = Arcsin x , puis t = tan θ , donnent :

dx

xxx

xx

.)

11(

11

0∫−+

− = 2 ∫ +2/

0 tan1π

θθd = 2 ∫

+∞

++0 )1²)(1( ttdt = ... = ln

²11

tt

++ + Arctan t +∞

0 = 2π .

Variante : le changement de variable t = x

x−1 donne le même résultat.

7) f(x) = )1()2(

1xxx −+

est continue positive sur ]0, 1[, équivalente à x2

1 en 0+ et à x−13

1 en

1−, donc intégrable sur ]0, 1[. Le changement de variable 2x – 1 = sin θ (plutôt θ = Arcsin… ) donne

I = ∫ −+1

0 )1()2( xxxdx = ∫ −−+

1

0 )²12(1)2(2

xxdx = ∫

+

− +2/

2/ sin5.2π

π θθd , puis t = tan(θ/2)…

> f:=x->1/((x+2)*sqrt(x*(1-x)));int(f(x),x);int(f(x),x=0..1); 16

6

arctan112

( )− + 2 5x 6

− + + ( ) + x 2 2 5 x 4

16

π 6

Exercice 14 : Convergence et calcul de

∫ −+a

xaxdxx

0 ²)²)(1²(. ( a > 0 ) , ∫

+∞

∞− ++ ²²²)²( bxaxdx ( a, b > 0 ) , ∫

+∞

+0 cos chxadx .

Solution (partielle) :

Page 12: Exercices intégrales généralisées

12

1) Le changement de variable y = x2 ramène la 1ère intégrale à I = ∫ −+

²

0 )²)(1(2

a

yaydy

.

L’intégrale est impropre en a2, mais convergente (règle de l’équivalent).

3) Si a ≡ π (mod 2π), ∫+∞

−0 1chxdx est impropre en 0 et en +∞, et divergente sur ]0, 1] (règle de

l’équivalent). Sinon, l’intégrale est impropre en +∞, et convergente (règle de l’équivalent).

∫+∞

+0 cos chxadx = ∫

∞+

++0 2 1cos2.2

aeedxe

xx

x

= ∫+∞

++1 2 1cos2.2

attdt = ∫

+∞

++1 ²sin)²cos(.2

aatdt ( où t = exp x )

Puis l’on pose t + cos a = u.sin a . On trouve au final a

asin

, si 0 < a < 2π .

Exercice 15 : Existence et calcul des intégrales : I(a) = ∫1

0.ln. dxxxa .

Solution : f(x) = xa.ln x est continue négative sur ]0, 1] .

Si 0 < a , f(x) → 0 en 0+ ; il y a fausse impropreté.

Si −1 < a , f(x) = O(2

11

a

x− ) au V(0+), car 2

1+a

x ln x → 0. Or 2

11

a

x− est intégrable sur ]0, 1].

Si a = − 1, f(x) = xxln est non intégrable, car ∫

1.ln

εdx

xx = −

2²ln ε → −∞. A fortiori si a < − 1.

Conclusion : I(a) est définie pour a > −1. Une intégration par parties donne alors I(a) = −)²1(

1+a

.

NB : le chgt de variable x = e−t

donne I(a) = ∫+∞ +−0

)1( .dtte ta , fournissant un autre angle d’attaque.

Exercice 16 : Existence et calcul de I = ∫2/

0).ln(sin

πdtt ( On pourra noter que I = ∫

2/

0).ln(cos

πdtt ).

Solution :

La fonction f : t → ln(sin t) est continue négative sur ]0, 2π ].

Au V(0+), f(t) = ln( t + O(t3) ) = ln t + O(t

2) ∼ ln t, donc f est intégrable.

Pour calculer I, notons que :

I = ∫2/

0).ln(sin

πdtt = ∫

2/

0).ln(cos

πdtt =

21 ∫

2/

0).cos.ln(sin

πdttt =

21 ∫

2/

0.

2)2sin(ln

πdtt

=

= 21 ∫

2/

0).2ln(sin

πdtt −

4π ln 2 =

41 ∫

π

0).ln(sin duu −

4π ln 2 =

21 ∫

2/

0).ln(sin

πduu −

4π ln 2

= 21 I −

4π ln 2 , par pliage, donc I = −

2π ln 2 .

Remarque : d’autres méthodes existent, moins astucieuses.

Exercice 17 : Nature de l’intégrale de Gauss ∫+∞

−0

².dxe x .

Solution : La fonction x → ²xe− est continue positive, mais ne se primitive pas élémentairement.

On ne peut donc montrer la convergence de l’intégrale en calculant ∫ −A

x dxe0

². . Nous allons procéder

par comparaison à des fonctions tests connues.

Page 13: Exercices intégrales généralisées

13

1ère méthode : je dis que, pour x assez grand, x ≥ a > 0, 0 ≤ ²xe− ≤ ²

1x

, car ²2 xex − → 0 quand x → +∞.

Comme ²

1x

est intégrable sur [a, +∞[, ²xe− itou.

2ème méthode : je dis que, pour tout x, 0 ≤ ²xe− ≤ 1²

1+x

, car 1 + x2 ≤ ²xe− (au fait, pourquoi ?).

Comme 1²

1+x

est intégrable sur [0, +∞[, ²xe− itou.

3ème méthode : je dis que, pour x ≥ 1, 0 ≤ ²xe− ≤ xe− . Comme xe− est intégrable sur [1, +∞[, ²xe− itou.

Remarque : On peut néanmoins calculer par divers moyens indirects l’intégrale de Gauss

∫+∞

−0

².dxe x , et démontrer qu’elle vaut 2π . La méthode la plus classique consiste à considérer

l’intégrale double ∫∫ +∞−−

[²,0[

²² .dxdye yx et à la calculer de deux façons. Mais on peut aussi passer par la

variable complexe.

Exercice 18 : Nature de l’intégrale ∫1

0).1sin( dx

x.

Solution : La fonction x → sin(1/x) est définie sur R*, et n’a pas de limite en 0, car elle oscille au V(0). L’intégrale proposée est impropre en 0.

Faute de pouvoir calculer ∫1

).1sin(ε

dxx

, on va montrer l’absolue convergence par comparaison.

Le plus simple est de noter que | sin x1 | ≤ 1. Comme la fonction x → 1 est intégrable sur ]0, 1], la

fonction x → sin(1/x) l’est également.

On peut aussi noter que le changement y = 1/x transforme l’intégrale en ∫+∞

1.

²sin dy

yy

, qui est

clairement absolument convergente.

Remarques : 1) On peut montrer qu’une fois prolongée arbitrairement en 0, la fonction x → sin(1/x) n’est pas réglée, mais est Riemann-intégrable sur [0, 1].

2) Maple affirme que ∫1

0).1sin( dx

x = sin 1 – Ci(1) ≈ 0.504. Encore faut-il connaître la fonction

« Cosinus intégral » Ci… > A:=int(sin(1/x),x=0..1);evalf(A);

:= A − ( )sin 1 ( )Ci 1 .5040670619

3) L’intégrale ∫+∞

0.1sin dx

x diverge car 0 < sin(1/x) ∼ 1/x au V(+∞).

Exercice 19 : Nature de l’intégrale ∫+∞

−0

).1cos1( dxx

.

Solution : La fonction x → 1 – cos(1/x) est continue positive sur ]0, +∞[.

Elle est bornée sur ]0, 1], et O(1/x2) au V(+∞), donc intégrable sur [1, ∞[.

En résumé, elle est intégrable sur ]0, +∞[.

Remarques : 1) Le changement de variable y = 1/x transforme l’intégale en ∫+∞ −0

cos1 dyy

y, qui est

faussement impropre en 0 et convergente sur [1, +∞[, car 0 ≤ ²

cos1y

y− ≤

²2y

.

2) Maple affirme que :

Page 14: Exercices intégrales généralisées

14

> B:=int(1-cos(1/x),x=0..infinity);evalf(B);

:= B12

π

1.570796327

Exercice 20 : Nature de l’intégrale A = ∫∞+

+0.

)1ln(²)/1sin(.

dxx

xx .

Solution : [ Oral Mines 1980 ]

C’est l’intégrale impropre en 0+ et en +∞ de la fonction f(x) = )1ln(²)/1sin(.

xxx

+ .

Etudions séparément son intégrabilité au V(+∞) et au V(0+).

• Au V(+∞), f(x) ∼ xx ln.

12/3 ≤ 2/3

1x

; par conséquent, f est intégrable.

• Au V(0+), | f(x) | ≤ )1ln( x

x+ ∼

x1 ; par conséquent, f est intégrable.

En fait, au V(0+), la fonction f(x) oscille entre )1ln( x

x+ et −

)1ln( xx+ .

> with(plots):f:=x->sqrt(x)*sin(1/x^2)/ln(1+x);g:=x->sqrt(x)/ln(1+x); > p:=plot(f(x),x=0..3,-4..4,thickness=2,numpoints=1000): q:=plot([g(x),-g(x)],x=0..3,-5..5,color=blue):display(p,q);

Exercice 21 : Discuter la nature des intégrales de Bertrand : ∫+∞

2 ).(ln ba xxdx et ∫

2/1

0 ln.b

a xxdx .

Solution : La première de ces intégrales est au programme sans être au programme, tout en l’étant…

• Notons F(a, b) la première intégrale. Elle converge ssi a > 1, ou (a = 1 et b > 1), autrement dit ssi (a, b) > (1, 1) pour l’ordre lexicographique.

C’est l’intégrale impropre en +∞ de la fonction continue et positive x → ba xx ).(ln1 .

Le plus simple est de commencer par le cas a = 1. Le changement de variable u = ln x donne :

∫A

bxxdx

2 ).(ln = ∫

)ln(

)2ln(

A

budu . On sait que cette intégrale converge ssi b > 1.

Si a > 1, je dis que 0 < ba xx ).(ln1 < 2).(ln

1xx

pour x assez grand, car xa−1

(ln x)b−2

→ +∞ en +∞.

Page 15: Exercices intégrales généralisées

15

Comme ∫+∞

2 2).(lnxxdx converge, ∫

+∞

2 ).(ln ba xxdx converge.

Si a < 1, je dis que 0 < x1 < ba xx ).(ln

1 pour x assez grand, car xa−1

(ln x)b → 0 en +∞.

Comme ∫+∞

2 xdx diverge, ∫

+∞

2 ).(ln ba xxdx diverge.

• Soit G(a, b) la seconde intégrale. Elle converge ssi a < 1 ou a = 1 et b > 1.

Le changement de variable u = 1/x donne en effet : ∫2/1

0 ln.b

a xxdx = ∫

+∞

−2 2 ln. uudu

ba .

Nous voilà ramenés aux intégrales précédentes. Mais on pouvait aussi rester au V(0+).

Remarque : l’intégrale ∫+∞

0 ln.b

a xxdx est toujours divergente. Elle est impropre en 0+, 1 et +∞.

En effet, en vertu de ce qui précède, il y a convergence sur ]0, ½] et [2, +∞[ ssi a = 1 et b > 1.

Mais au V(1), ba xx ln.1 ∼ b

x 11−

. Il y a convergence sur [½, 2] ssi b < 1.

Exercice 22 : Discuter la nature de l’intégrale : ∫+∞

3 )ln.(ln).(ln cba tttdt .

Solution :

• Soit F(a, b, c) la première intégrale. Elle converge ssi a > 1, ou a = 1 et b > 1, ou a = b = 1 et c > 1, autrement dit ssi (a, b, c) > (1, 1, 1) pour l’ordre lexicographique. Le plus simple est d’étudier le cas a = b = 1. Le chgt de variable u = ln ln t donne

∫+∞

3 )ln.(lnln. ctttdt = ∫

+∞

)3ln(ln cudu . L’intégrale converge ssi c > 1.

Si a > 1, ou a = 1 et b > 1, alors fa,b,c(t) ≤ f1,1,2(t) au V(+∞).

Si a < 1, ou a = 1 et b < 1, alors fa,b,c(t) ≥ f1,1,1(t) au V(+∞).

• Soit G(a, b) la seconde intégrale. Elle converge ssi a < 1 ou a = 1 et b > 1.

Le changement de variable u = 1/t donne en effet : ∫2/1

0 ln.b

a ttdt = ∫

+∞

−2 2 ln. uudu

ba .

Nous voilà ramenés à des Bertrand classiques. Mais on peut aussi rester au V(0+).

Exercice 23 : Discuter la nature des intégrales : ∫+∞

+1 )1.( ba xxdx , ∫

+∞

+0 )1.( ba xxdx et ∫

∞+ +0

.)1ln(

dtt

tb

a

.

Solution :

1) La fonction f(x) = )1(

1ba xx + est continue et positive sur [1, +∞[ .

En +∞, f(x) ∼ bax +1 si b > 0 ; il y a intégrabilité ssi a + b > 1.

f(x) ∼ ax21 si b = 0 et f(x) ∼ ax

1 si b < 0 ; il y a intégrabilité ssi a > 1.

I(a, b) = ∫+∞

+1 )1.( ba xxdx est définie sur D = (a, b) ; b > 0 et a + b > 1 ∪ (a, b) ; b ≤ 0 et a > 1 .

Page 16: Exercices intégrales généralisées

16

2) En 0+, f(x) ∼ ax1 si b > 0 et f(x) ∼ ax2

1 si b = 0 ; il y a intégrabilité ssi a < 1.

f(x) ∼ bax +1 si b < 0 ; il y a intégrabilité ssi a + b < 1.

J(a, b) = ∫+∞

+0 )1.( ba xxdx est définie sur D ∩ D’, avec :

D’ = (a, b) ; b ≥ 0 et a < 1 ∪ (a, b) ; b < 0 et a + b < 1 .

Exercice 24 : 1) Soit f ∈ C(R+, R+) une fonction intégrable. Construire une suite croissante (xn)

telle que (∀n) ∫+∞

nxdttf ).( ≤ n2

1 . En déduire qu’existe une fonction g ∈ C(R+, R+) croissante,

tendant vers +∞ en +∞, telle que f.g soit intégrable sur R+.

2) Soit f ∈ C(R+, R+) une fonction non intégrable. Montrer qu’il existe une fonction g∈C(R+, R+)

décroissante, tendant vers 0 en +∞, telle que f.g ne soit pas intégrable sur R+.

Solution :

2. Intégrales généralisées et séries.

Exercice 1 : Soit ∑+∞

=0n

na une série à termes ≥ 0.

Montrer que la fonction f : R+ → R+ définie par f(x) = an pour n ≤ x < n+1, est intégrable sur R+ si

et seulement si la série ∑+∞

=0n

na converge, et qu’alors : ∫+∞

0).( dxxf = ∑

+∞

=0n

na .

Solution :

Exercice 2 : 1) Indiquer une fonction continue positive et intégrable sur R+, bornée mais ne tendant pas vers 0 en +∞.

2) Indiquer une fonction continue positive intégrable et non bornée sur R+.

3) Montrer toutefois que si f est continue, positive et intégrable sur R+ et intégrable, 0 est valeur

d’adhérence de f en +∞, autrement dit, il existe une suite (xn) tendant vers +∞ telle que f(xn) → 0.

Solution : Considérons la fonction continue affine par morceaux f : R+ → R+ définie par :

f(n) = n , f(n ± nn 2.1 ) = 0 (n ≥ 1) , f étant affine entre ces points.

Alors ∫+ 2/1

0).(

ndttf =

21 +

²21 + ... + n2

1 = 1 − n21 ↑ 1.

Comme tout segment [a, b] est inclus dans un [0, n +21 ], f est intégrable sur R+, d’intégrale 1.

Exercice 3 : a) Nature de l’intégrale ∫+∞

+0 2 ²sin.1 xxdx .

Page 17: Exercices intégrales généralisées

17

b) Soient a et b > 0. Montrer que l’intégrale ∫∞+

+0 ²sin.1.

xxdxx

b

a

converge ssi b > 2a + 2.

Solution :

1) Analyse. La fonction f(x) = xx ²sin².1

1+ est continue positive sur R+.

L’encadrement ²1

1x+ ≤ f(x) ≤ 1 ne conclut pas.

Le graphe de f oscille de l’une à l’autre fonction, donc f n’a pas d’équivalent simple en +∞.

> with(plots):f:=x->1/(1+x^2*sin(x)^2): p:=plot(f(x),x=0..15,thickness=2,numpoints=2500): q:=plot(1,x=0..15,color=black):r:=plot(1/(1+x^2),x=0..15,color=blue): display(p,q,r);

2) Transformons l’intégrale en série : on sait en effet que f intégrable si et seulement si la série

∑+∞

=0n

nu converge, où un = ∫+ π

π

)1().(

n

ndxxf = ∫ ++

π

π0 ²sin)²(1 unudu (x = nπ + u).

Encadrons un : ∫ ++π

π0 ²sin².)²1(1 undu ≤ un ≤ ∫ +

π

π0 ²sin².²1 undu .

an = ∫ +π

π0 ²sin².²1 undu = 2∫ +

2/

0 ²sin².²1π

π undu se calcule aisément, la règle de Bioche autorisant t = tan u.

an = 2∫+∞

+++0 ]²1²²²1²)[1(

ttnt

dtπ = 2∫

+∞

++0 ²²).²1(1 tndt

π = ²²1 π

πn+

∼ n1 .

Par encadrement, un ∼ n1 , donc ∑

+∞

=0n

nu et ∫+∞

+0 2 ²sin.1 xxdx divergent.

Variante : évitons le calcul exact de an en utilisant l’encadrement ∀u∈[0, 2π ] π

u2 ≤ sin u ≤ u.

2∫ ++2/

0 ²²)²1(1π

π undu ≤ an = 2∫ +

2/

0 ²sin².²1π

π undu ≤ 2∫ +

2/

0 ²²41π

undu .

Un calcul montre que (an) est semblable à n1 . Résultat plus faible, mais qui suffit à conclure.

3) Généralisation facile laissée au lecteur.

Références : Valiron p. 114, Ramis p. 102, Couty-Ezra, p. 401, Ovaert 2 p. 630, POX 1984, etc.

Exercice 4 : Soient a, b > 0. Montrer que l’intégrale ∫+∞

−0

).²sin.exp(. dxxxx ba converge ssi b > 2a + 2.

Solution :

1) Analyse. La fonction f(x) = xa.exp(−x

b sin

2 x) est continue et positive sur R+.

On a l’encadrement xa.exp(−x

b) ≤ f(x) ≤ x

a , le graphe de f oscillant de l’une à l’autre.

> with(plots):f:=x->x*exp(-x^3*sin(x)^2);

Page 18: Exercices intégrales généralisées

18

p:=plot(f(x),x=0..15,numpoints=2500,thickness=2): q:=plot(x,x=0..15,color=black): r:=plot(x*exp(-x^3),x=0..15,color=blue):display(p,q,r);

2) Transformons l’intégrale en série : on sait en effet que f intégrable si et seulement si la série

∑+∞

=0n

nu converge, où un = ∫+ π

π

)1().(

n

ndxxf = ∫ +−+

πππ

0).²sin.)(exp(.)( duuunun ba ( x = nπ + u ).

Encadrons un : ∫ +−π

ππ0

).²sin.))1((exp(. duunn baa ≤ un ≤ ∫ −+π

ππ0

).²sin.)(exp(.)1( duunn baa .

an = ∫ −π

π0

).sin..)(exp( duun b = 2∫ −2/

0).²sin..)(exp(

ππ duun b . Or ∀u ∈ [0,

2π ] π

u2 ≤ sin u ≤ u.

Donc 2∫ −2/

0²)..)(exp(

ππ duun b ≤ an ≤ 2∫ −

2/

0).

²²4.)(exp(

π

ππ duun b .

Le chgt de variable s = u(nπ)b/2

donne ∫ −2/

0²)..)(exp(

ππ duun b = 2/)(

1bnπ ∫ −

2/)(

0

²2/

.ππ bn

s dse ∼ 2/)(2 bnππ .

On traite de même la majorante. On en déduit que (an) est semblable à 2/1bn

.

Comme (nπ)a

an+1 ≤ un ≤ ((n+1)π)a an , (un) est semblable à abn −2/

1 .

Finalement, ∑+∞

=0n

nu converge ssi b > 2a + 2.

Remarques : 1) La méthode de Laplace fournit même un équivalent de un, car elle affirme que :

∫ −2/

0).²sin..)(exp(

ππ duun b ∼ ∫ −

2/

0²)..)(exp(

ππ duun b ∼ 2/)(2 bnπ

π .

2) Comme xa.exp(−x

b sin

2 x) ≥ x

a.exp(−x

b | sin x |), la condition b > 2a + 2 assure la convergence de

∫+∞

−0

).sin.exp(. dxxxx ba . L’exercice suivant creuse cette question.

Exercice 5 : i) Nature de l’intégrale ∫+∞

−0

).sin.exp( dxxx .

ii) Soient a, b > 0. Montrer que l’intégrale ∫+∞

−0

).sin.exp(. dxxxx ba converge ssi b > a + 1.

Solution : 1) Analyse. C’est l’intégrale impropre en +∞ de la fonction continue et positive f(x) = exp( − x.| sin x | ). f est une fonction oscillante, car (∀x) exp(−x) ≤ f(x) ≤ 1, le graphe de f oscillant constamment entre ces deux fonctions.

Page 19: Exercices intégrales généralisées

19

> with(plots): > p:=plot(1,0..15,color=black):q:=plot(exp(-x),x=0..15,color=blue): r:=plot(exp(-x*abs(sin(x))),x=0..15,numpoints=2000,thickness=2): display(p,q,r);

2) Transformons l’intégrale en série : on sait en effet que f est intégrable si et seulement si la série

∑+∞

=0n

nu converge, où un = ∫+ π

π

)1().(

n

ndxxf = ∫ +−

ππ

0).sin).(exp( duuun (x = nπ + u).

Encadrons un : 0 ≤ ∫ +−π

π0

).sin..)1(exp( duun ≤ un ≤ ∫ −π

π0

).sin..exp( duun .

an = ∫ −π

π0

).sin..exp( duun = 2∫ −2/

0).sin..exp(

ππ duun . Or ∀u ∈ [0,

2π ] π

u2 ≤ sin u ≤ u.

Donc 2∫ −2/

0).exp(

ππ duun ≤ an ≤ 2∫ −

2/

0).2exp(

πdunu .

Ces integrales se calculent et montrent que (an) est semblable à n1 .

Comme an+1 ≤ un ≤ an , (un) est également semblable à n1 .

Finalement, la série et l’intégrale divergent.

Remarque : la méthode de Laplace fournit même un équivalent de an = 2∫ −2/

0).sin..exp(

ππ duun .

Par concentration de masse en 0, on montre que an ∼ 2∫ −2/

0).exp(

ππ duun ∼ πn

2 .

3) Généralisation. La fonction f(x) = xa.exp(−x

b | sin x |) est intégrable si et seulement si la série

∑+∞

=0n

nu converge, où un = ∫+ π

π

)1().(

n

ndxxf = ∫ +−+

πππ

0).sin.)(exp(.)( duuunun ba ( x = nπ + u ).

Encadrons un : ∫ +−π

ππ0

).sin.))1((exp(. duunn baa ≤ un ≤ ∫ −+π

ππ0

).sin.)(exp(.)1( duunn baa .

an = ∫ −π

π0

).sin..)(exp( duun b = 2∫ −2/

0).sin..)(exp(

ππ duun b . Or ∀u ∈ [0,

2π ] π

u2 ≤ sin u ≤ u.

Donc 2∫ −2/

0)..)(exp(

ππ duun b ≤ an ≤ 2∫ −

2/

0).2.)(exp(

π

ππ duun b .

Ces integrales se calculent et montrent que (an) est semblable à bn1 .

Comme (nπ)a

an+1 ≤ un ≤ ((n + 1)π)a an , (un) est semblable à abn −

1 .

Finalement, ∑+∞

=0n

nu converge ssi b > a + 1.

Exercice 6 : Natures des intégrales :

∫∞+

− +0 3

2

sin.

.

xee

dxexx

x

, ∫+∞

0.sin dxx

x , ∫

+∞

πdx

xxx.

ln.²sin , ∫

+∞−

0. atet E(et).dt (E est la partie entière) .

Page 20: Exercices intégrales généralisées

20

Solution :

Exercice 7 : Natures des intégrales : ∫∞+ −

0 cos

.

t

dte t

, ∫+∞

+0 3/2sin²).1( xxdx , ∫

+∞

π ba xxdxsin.

, (a, b) ∈ R2.

Solution : Etudions en détail la première intégrale, tirée de Arnaudiès-Fraysse, t.2, p. 424.

La fonction f(t) = )cos(t

e t− est continue > 0 sur [0,

2π [ ∪ ]

2π ,

23π [ ∪ …

Cette fonction possède une infinité d’asymptotes sur R+ ( Ah ! posséder une asymptote !…)

Donc l’intégrale est impropre en tous les nπ + 2π et bien sûr en +∞.

Chacune des intégrales In = ∫+

−2

2

).(ππ

ππ

n

ndttf converge en vertu de la règle de l’équivalent.

De plus, le changement de variable t = nπ + u donne In = e−nπ∫

+

−2

2

).(π

π dttf .

Enfin, la série de terme général In converge en tant que série géométrique.

Par conséquent, I = ∫+

+2

0).(

πdttf + ∑

1n

ne π ∫+

−2

2

).(π

π dttf = ∫+

+2

0).(

πdttf +

11−πe ∫

+

−2

2

).(π

π dttf .

Remarque : la fonction F(x) = ∫x

dttf0

).( est intéressante. Elle est croissante, de limite I en +∞, C∞

en presque tout point, mais ayant des tangentes verticales en tous les nπ + 2π .

> with(plots):f:=t->exp(-t)/sqrt(abs(cos(t))); A:=int(f(t),t=-Pi/2..Pi/2);B:=int(f(t),t=0..Pi/2); C:=evalf(B+A/(exp(Pi)-1));

:= C 1.481451160 > p:=plot(f(t),t=0..3*Pi,0..4,color=blue,numpoints=10000,thickness=2): q:=plot(exp(-t),t=0..3*Pi):display(p,q); > F:=x->int(f(t),t=0..x);plot([C,F(x)],x=0..3*Pi,thickness=2);

Exercice 8 : Lemme de Riemann-Lebesgue.

Soient f : I → K ou E une fonction intégrable, p : R → K une fonction réglée T-périodique.

Montrer que : limn→+∞ ∫I f(t).p(nt).dt = T1 ∫

Tdttp

0).( . ∫I f(t).dt .

Application : Montrer que la fonction de Bessel J0(x) = π2 ∫

2/

0).sin.cos(

πθθ dx tend vers 0 en ±∞.

[Poser sin θ = t].

Page 21: Exercices intégrales généralisées

21

Solution :

Exercice 9 : Soit f : R → R continue et bornée. Montrer que ∫+∞

∞− +− dt

tttf

²11)

21²( = ∫

+∞

∞− + dtttf²1)(

.

Solution :

Exercice 10 : Formule de Schlömilch . Soient f : I = ]0, +∞[ → C réglée sur tout segment, a et c > 0.

On suppose que l’intégrale ∫I f((cx − xa )2).dx converge.

Montrer que l’intégrale ∫I f(t2).dt converge, et que : ∫I f((cx − xa )2).dx =

c1 .∫I f(t2).dt .

Solution : Schlömich est un mathématicien allemand, dont le nom est plus facile à prononcer quand on a une pomme de terre chaude dans la bouche.

Exercice 11 : Soient p1, p2, …, pn des réels > 0, a1 < a2 < … < an.

1) Etudier les variations de ϕ(x) = x − ∑= −

n

j j

j

axp

1

.

2) Soit y ∈ R. Combien l’équation ϕ(x) = y a-t-elle de solutions en x ? Quelle est leur somme ?

3) Soit f ∈ C(R, R) intégrable. Montrer que ∫+∞

∞−dttf )).((ϕ = ∫

+∞

∞−dttf ).( .

4) Applications : Calculer ∫+∞

∞−−− dt

tpt ).

²2²

2²exp( (p > 0), puis ∫

+∞− −−

0

2/3 ).exp( dttp

stt (s et p >0).

Solution : Problème 12 : intégrale de fractions rationnelles.

1) Soit F(x) = )()(

xQxP ∈ C(X) une fraction rationnelle sous forme réduite.

F est intégrable sur R si et seulement si F n’a pas de pôle réel, et deg Q ≥ deg P + 2.

Et alors : dxxQxP

.)()(

∫+∞

∞− = 2iπ. ∑

+Π∈a

a Fs )(Re = −2iπ. ∑−Π∈a

a Fs )(Re .

où Π (resp. Π+, Π−) est l’ensemble des pôles de F (resp. dans Im z > 0, dans Im z < 0) et Resa(F) est

le résidu de F en a (c’est-à-dire le coef. de az−

1 dans la décomposition en éléments simples de F).

2) Applications :

i) Calculer ∫+∞

∞− +14xdx et ∫

+∞

∞− ++ 1²4 xxdx .

ii) Montrer que ∫+∞

∞− +12nxdx =

)/sin(. nn ππ et ∫

∞+

∞− + dxxxn

p

.12

2

= )

212

sin(. ππ

np

n+ ( p < n ).

iii) Calculer ∫+∞

∞− + nxdx

)1( 2 et F(a) = ∫

+∞

∞− +−+ )1cos2²)².(1( 2 axxxdx .

Solution :

Page 22: Exercices intégrales généralisées

22

3. Intégrales impropres et sommes de Riemann. La théorie des sommes de Riemann s’étend-elle aux intégrales généralisées ? La réponse est évidemment oui ( il y a des théorèmes ), mais le programme est muet sur ce sujet. Il faut donc se débrouiller à la main.

Exercice 1 : Trouver limn→+∞ ∑−

= −

1

0 ²²1

n

k kn.

Solution : Sn = ∑−

= −

1

0 ²²1

n

k kn =

n1 ∑

= −

1

0 )²/(11

n

k nk est une somme de Riemann de f(x) =

²11

x−

associée à la subdivision ( 0, n1 , …,

nn 1− ). Elle tend vers ∫

1

0).( dxxf =

2π …

Mais il y a un problème ! Nous n’avons jamais démontré de théorème concernant la convergence des sommes de Riemann dans le cas des intégrales impropres. Pour justifier rigoureusement ce passage à la limite, on procèdera par un encadrement intégral basé sur la croissance de la fonction f sur [0, 1[ :

n1 + ∫

− ndxxf

/11

0).( ≤ Sn =

n1 ∑

= −

1

0 )²/(11

n

k nk ≤ ∫

1

0).( dxxf .

Il reste à faire tendre n vers l’infini, à l’aide du lemme des gendarmes.

Exercice 2 : Trouver limn→+∞ ∑−

= −+

1

0 ².)1(²1

n

kk kn

.

Solution : [ Oral Mines PC 2012, RMS n° 697 ]

Sn = ∑−

= −+

1

0 ².)1(²1

n

kk kn

= ∑= +

M

p pn0 ²4²1 + ∑

= +−

N

p pn0 )²12(²1 , où M = [

21−n ] et N = [

2n ] − 1.

La première somme se ramène à des sommes de Riemann classiques :

An = ∑= +

M

p pn0 ²4²1 =

nM

M1 ∑

= +

M

p

np0

²²41

1 → 21 ∫ +

2/1

0 ²41 xdx =

41 Argsh 1.

La seconde somme se ramène à des sommes de Riemann voisines de celles de l’exercice précédent.

Bn = ∑= +−

N

p pn0 )²12(²1 =

nN

N1 ∑

= +−

N

p

np0

²)²12(1

1 → 21 ∫ −

1

0 ²1 xdx =

4π .

Conclusion : Sn = ∑−

= −+

1

0 ².)1(²1

n

kk kn

→ 41 Argsh 1 +

4π .

Exercice 3 : 1) Donner un exemple de fonction réglée f : ]0, 1] → R telle que ∫1

0).( dxxf converge

sans que la suite un = n1 ∑

=

n

k nkf

1

)( converge.

2) Montrer que si f : ]0, 1] → R est réglée et bornée, alors l’intégrale I = ∫1

0).( dxxf converge, et la

suite un = n1 ∑

=

n

k nkf

1

)( converge vers I quand n tend vers l’infini.

Page 23: Exercices intégrales généralisées

23

3) Soit f : ]0, 1[ → R monotone. Montrer les résultats suivants :

a) ∫1

0).( dxxf converge ⇒ un → ∫

1

0).( dxxf ;

b) La convergence de (un ) n’implique pas celle de ∫1

0).( dxxf ( considérer f(x) =

x1 −

x−11 ) ;

c) Si f est bornée en 0 ou en 1, la convergence de (un ) implique celle de ∫1

0).( dxxf .

Solution :

Exercice 4 : Montrer que limn→+∞ ∏−

=

1

1

1(n

k

− nk )1/k = exp(−

6²π ) .

Solution : Passant au log, tout revient à montrer que :

∑−

=−

1

1

)1ln(1n

k nk

k → −

6²π , ou encore que Sn =

n1 ∑

=−

1

1

)1ln(n

k nk

kn → −

6²π .

Or Sn est une somme de Riemann de la fonction f(x) = x

x)1ln( −.

Elle tend donc vers ∫1

0).( dxxf = − ∫∑

+∞

=

−1

0 1

1

.n

n

dxn

x = −∑∫

+∞

=

1

1

0

1

.n

n

dxn

x = −∑

+∞

=1 ²1

n n = −

6²π .

Il reste cependant deux points à préciser : • La fonction f est négative, intégrable et décroissante sur [0, 1[, après prolongement en 0 (étude des varaitions, ou développement en série); la convergence des sommes de Riemann reste encore vraie. • Le théorème d’intégration terme à terme des séries s’applique. Exercice 5 : Lemmes de pincement.

1) Soit f : ]0, +∞[ → R+ une fonction décroissante et intégrable. Montrer que, pour tout h > 0, la

série ∑+∞

=1

)(n

nhf converge, et que : lim h→0+ h∑+∞

=1

)(n

nhf = ∫+∞

0).( dttf .

2) Soit f : [0, +∞[ → R une fonction réglée, décroissante sur [A, +∞[, et intégrable. Montrer que,

pour tout h > 0, la série ∑+∞

=1

)(n

nhf converge, et que : lim h→0+ h∑+∞

=1

)(n

nhf = ∫+∞

0).( dttf .

On pourra écrire h.∑

+∞

=1

)(n

nhf = h.∑=

N

n

nhf1

)( + h ∑+∞

+= 1

)(Nn

nhf , où N = [hA ] .

3) Soit f : ]0, +∞[ → E (Banach) une fonction réglée. On suppose qu’existe g : ]0, +∞[ → R

décroissante et intégrable, telle que (∀x > 0) || )(xf || ≤ g(x). Montrer que f est intégrable, que, pour

tout h > 0, la série ∑+∞

=1

)(n

nhf converge, et que : lim h→0+ h∑+∞

=1

)(n

nhf = ∫+∞

0.)( dttf .

Solution : 1) Rappelons qu’une fonction décroissante est réglée sur tout segment.

On a l’encadrement intégral : ∫+ hN

hdxxf

)1().( ≤ h.∑

=

N

n

nhf1

)( ≤ ∫Nh

dxxf0

).( ≤ ∫+∞

0).( dxxf .

La suite N → ∑=

N

n

nhf1

)( est croissante majorée, donc elle converge, et la série ∑+∞

=1

)(n

nhf converge.

Fixons h > 0 et faisons tendre N vers l’infini. Il vient : ∫+∞

hdxxf ).( ≤ h.∑

+∞

=1

)(n

nhf ≤ ∫+∞

0).( dxxf .

Page 24: Exercices intégrales généralisées

24

En vertu du lemme des gendarmes, h.∑+∞

=1

)(n

nhf tend vers ∫+∞

0).( dxxf quand h tend vers 0+.

2) Soit f : [0, +∞[ → R une fonction réglée, décroissante sur [A, +∞[, et intégrable.

Pour tout h > 0, la série ∑+∞

=1

)(n

nhf converge, pour la même raison qu’en 1).

Ecrivons h.∑

+∞

=1

)(n

nhf = h.∑=

N

n

nhf1

)( + h ∑+∞

+= 1

)(Nn

nhf , où N = [hA ] .

D’une part, h.∑=

N

n

nhf1

)( → ∫A

dxxf0

).( par un argument de sommes de Riemann.

D’autre part, h ∑+∞

+= 1

)(Nn

nhf → ∫+∞

Adxxf ).( par encadrement comme ci-dessus.

Exercice 6 : Montrer que la fonction f : x → sinx1 est intégrable sur ]0, 1].

En déduire que la fonction f(x) = sinx1 si x ≠ 0 , f(0) = 0 , admet une primitive sur R.

Solution : 1) f est continue sur ]0, 1] et bornée, donc intégrable.

Remarques : i) Si on la prolonge en 0 par f(0) = a , f est Riemann-intégrable et non réglée.

ii) Le changement de variable y = x1 donne ∫

1

0.sin dx

xx = ∫

+∞

1.

²sin dy

yy

.

2) Considérons la fonction F(x) = ∫x

dttf0

).( .

Elle est définie et continue sur R, dérivable et même C∞

sur R*, car F’(x) = sinx1 .

Reste à montrer que F est dérivable en 0 et que F’(0) = 0. Une intégration par parties donne :

F(x) = ∫x

dttf0

).( = ∫ −x

tdt

0)1(cos². = − t

2.cos

t1 | x

0 + 2∫x

dtt

t0

.1cos. = − x2.cos

x1 + 2∫

xdt

tt

0.1cos. .

En toute rigueur, il faudrait intégrer sur [ε, x], puis faire tendre ε vers 0. Cette formule est vraie pour tout x ≠ 0.

Elle reste vraie pour x = 0, car − x2.cos

x1 et 2∫

xdt

tt

0.1cos. tendent vers 0 quand x tend vers 0.

Par suite xxF )(

= − x.cosx1 +

x2 ∫

xdt

tt

0.1cos. = − x.cos

x1 +

x2 ∫

xdttg

0).( , où g est continue sur R.

Donc xxF )(

tend vers 0 et F’(0) = 0.

3) Voici une autre présentation : Les fonctions qui admettent une primitive sur R, c’est-à-dire les fonctions dérivées, forment un espace vectoriel, image de DDDD(R, R) par l’application D : y → y’. Cet espace vectoriel contient, on le sait, les fonctions continues.

Or (x2 cos

x1 )’ = 2x cos

x1 − sin

x1 . (x

2 cos

x1 )’ est une dérivée, 2x cos

x1 est continue sur R, après

prolongement en 0, donc est une dérivée. Leur différence f est aussi une dérivée. Cela montre que l’espace des fonctions primitivables contient strictement l’espace des fonctions continues.

Page 25: Exercices intégrales généralisées

25

4. Intégration des relations de comparaison.

Exercice 1 : Étudier F(x) = ∫−1

.x

t

dtt

e au V(0+).

Solution :

La fonction f(t) = t

e t−

est positive et non intégrable sur ]0, 1] car équivalente en 0+ à g(t) = t1.

On en déduit que F(x) tend vers +∞ quand x tend vers 0+ .

Le TIRC (IIc) donne aussitôt : F(x) ∼ ∫1

x tdt = − ln x (*)

mais on peut éviter d’appliquer le TIRC en écrivant : F(x) = ∫1

x tdt + ∫

−−1.

1x

t

dtt

e = − ln x + H(x) ,

où H(x) = ∫−−1

.1

x

t

dtt

e. La fonction h(t) =

te t 1−−

est prolongeable par continuité en 0 (et même

développable en série entière), de sorte que l’intégrale ∫]0,1] te t 1−−

.dt est faussement impropre.

Ainsi : F(x) = − ln x + c + o(1) , où c = ∫]0,1] te t 1−−

.dt . Résultat plus précis que (*) !

Exercice 2 : Intégrale de Frullani.

Convergence et calcul de ∫∞+ −− −

0.dt

tee btat

pour a et b > 0.

Solution : Bien que les fonctions intégrées ne se primitivent pas élémentairement, on peut calculer cette intégrale impropre.

La fonction f(t) = tee btat −− −

est continue sur ]0, +∞[ , prolongeable par continuité en 0 car

f(t) = t1(1 − at − 1 + bt + O(t

2)) = b − a + O(t) → b − a, de sorte que l’intégrale est faussement

impropre en 0. Enfin, sur [1, +∞[ f est différence de deux fonctions intégrables.

Attention, écrire : Ι = ∫∞+ −

0.dt

te at

− ∫∞+ −

0.dt

te bt

n’a pas de sens ... mais plutôt : I = limε→0+ I(ε) ,

où I(ε) = ∫∞+ −− −

εdt

tee btat

. = ∫∞+ −

εdt

te at

. − ∫∞+ −

εdt

te bt

. = ∫∞+ −

εa

u

duu

e. − ∫

∞+ −

εb

u

duu

e. = du

ueb

a

u

.∫−ε

ε

(chgts de var u = at et u = bt , puis Chasles) .

Or u

e u−

∼ u1 > 0 au V(0+) . Donc, par un argument d’intégration de relations de comparaison, qui

ne se déduit pas de (I, c), mais se montre comme (I, c), on a I(ε) ∼ ∫ε

ε

b

a udu = ln

ab .

Une variante élémentaire consiste à écrire : I(ε) = ∫ε

ε

b

a tdt + ∫

−−ε

ε

b

a

t

dtt

e.

1 = ln

ab + ∫

ε

ε

b

adttg ).( ,

où g(t) est continue sur R. Donc ε → ∫ε

ε

b

adttg ).( est continue sur R, et tend vers 0 en 0. cqfd.

Remarque : On donnera dans la suite (§ 9) une autre méthode de calcul, en considérant l’intégrale comme fonction de a.

Page 26: Exercices intégrales généralisées

26

Exercice 3 : Convergence et calcul de ∫ −1

0.

ln1dxx

x .

Solution : [ Oral X MP 2011, RMS n° 205 et Mines PC, n° 723 ] 1) S’assurer de la convergence de l’intégrale.

2) Le changement de variable s = − ln x, x = exp(−s) donne aussitôt :

I = ∫+∞ −− −0

2

.dttee tt

. C’est une Frullani, qui vaut ln 2.

3) On peut aussi noter que : I = limε→0+ ∫−ε1

0.

lndx

xx − ∫

−ε1

0 lnxdx ,

puis faire le changement de variable y = x2 dans la première intégrale.

I = limε→0+ ∫−

−−

ε

ε

1

)²1( lnxdx , et conclure par un argument d’intégration de relations de comparaison.

Exercice 4 : 1) Etudier la fonction F(x) = ∫∞+ −

x

t

dtte

au V(0+) .

2) Convergence et calcul de : ∫∞+ −− −

0)².( dt

tee btat

, pour a et b > 0.

Solution : On montre aisément que f(t) = (tee btat −− −

)2 est intégrable sur ]0, +∞[. Développons :

J(ε) = ∫+∞ −− −

εdt

tee btat

)².( = ∫∞+ −

εdt

te at

2

− 2∫∞+ +−

εdt

te tba

)(

+ ∫∞+ −

εdt

te bt

2

= 2a∫∞+ −

εa

u

duue

2.

² − 2( a + b ) ∫

∞+

+

ε)(.

²ba

u

duue

+ 2b∫∞+ −

εb

u

duue

2.

².

en faisant les changements de variables respectifs u = 2at , u = (a + b)t et u = 2bt.

Notons F(x) = ∫∞+ −

x

u

duue

, de sorte que J(ε) = 2a.F(2aε) − 2(a + b) F((a + b)ε) + 2b.F(2bε) .

Il s’agit d’obtenir un développement asymptotique de F(x) au V(0+). Or écrivons

F(x) = ∫+ −1

1x

duu

u + ∫+ − +−1

.²1

x

u

duu

ue + ∫

∞+ −

1.

²du

ue u

.

= x1 + ln x + 1 + ∫

+ − +−1

0.

²1

duu

ue u

+ ∫∞+ −

1.

²du

ue u

+ o(1) = x1 + ln x + C + o(1) .

car l’intégrale ∫+ − +−1

0.

²1

duu

ue u

est faussement impropre en 0. Finalement :

J(ε) = 2a ( εa21 + ln ε + ln(2a) + C + o(1))

− 2(a + b) ( ε)(1ba+ + ln ε + ln(a + b) + C + o(1))

+ 2b( εb21 + ln ε + ln(2b) + C + o(1)) = 2a ln(2a) – 2(a + b)ln(a + b) + 2b ln(2b) + o(1).

Si a, b > 0 , ∫∞+ −− −

0.dt

tee btat

= lnab et ∫

∞+ −− −0

)².( dttee btat

= 2a ln(2a) – 2(a + b)ln(a + b) + 2b ln(2b).

Maple confirme :

> F:=x->int(exp(-t)/t^2,t=x..infinity);F(x);series(F(x),x=0,6);

:= F → x d⌠

x

eeee( )−t

t2 t −− + eeee

( )−x( )Ei ,1 x x

x

Page 27: Exercices intégrales généralisées

27

Exercice 4 : Soit f : R+ → R+ continue intégrable. Pour x > 0, on pose g(x) = ∫x

xdttf

x2

).(1 .

Montrer que g est intégrable sur R*+ et calculer ∫+∞

0).( dxxg .

Solution : [ Oral Mines 2009, RMS n° 505 ]

La fonction g est de classe C1 sur ]0, +∞[. Et g(x) = ∫

2

1).( duxuf .

1) Formellement, I = ∫+∞

0).( dxxg = ∫ ∫

+∞

0

2

1.)).(( dxduxuf = ∫∫D dudxxuf ).( , où D = ]0, +∞[×[1, 2].

Intervertissons les intégrations : I = ∫ ∫+∞2

1 0.)).(( dudxxuf = ∫ ∫

+∞2

1 0.)).(1( dudttf

u = ln 2.∫

+∞

0).( dttf .

Tout cela est légitime en vertu du théorème de Fubini. L’intégrabilité de g passe par f +

et f −

.

2) Autre solution : Notons F(x) = ∫x

dttf0

).( ; alors g(x) = x

xFxF )()2( −.

La fonction F tend vers 0 en 0+, vers J = ∫+∞

0).( dttf en +∞ .

I = ∫+∞ −0

.)()2( dxx

xFxF est alors une intégrale de Frullani.

Sa convergence vers ln 2.∫+∞

0).( dttf se montre comme dans les exercices précédants.

Elle passe par : ∫−A

dxx

xFxFε

.)()2( = ∫

Adx

xxF2

2.)(

ε − ∫

Adx

xxF

ε.)(

= ∫A

Adx

xxF2

.)( − ∫

ε

ε

2.)( dx

xxF

,

et un argument d’intégration de relations de comparaison :

∫ε

ε

2.)( dx

xxF

→ 0 quand ε → 0 , et ∫A

Adx

xxF2

.)( → J.ln 2 quand A → +∞.

Exercice 5 : Trouver les parties principales au V(+∞) des fonctions :

F(x) = ∫ +x

dttt

tth0

.)1(

)( , F(x) = ∫ +

xt dt

t0.)11( , F(x) = ∫

x

edtt).ln(ln , F(x) = ∫

+∞

+x tdt

14 ,

F(x) = ∫ +x

x tdt2

4 1 , F(x) = ∫ ++

−xdt

ttt

0 4.

1²)1(²1 , F(x) = dt

tttx.

1ln.

1∫ + , F(x) = dtt

tx.

1²)exp(

1∫ + .

Solution :

♣ F(x) = ∫ +x

dttt

tth0

.)1(

)( ∼ ∫

x

tdt

1 = ln x. ♦ F(x) = ∫ +

xt dt

t0.)11( ∼ ∫

xdte

0. = ex.

♥ F(x) = ∫x

edtt).ln(ln ∼ x.ln ln x (I.P.P.) ♠ F(x) = ∫

+∞

+x tdt

14 ∼ ∫

+∞

x tdt²

= x1 .

♣ F(x) = ∫ +x

x tdt2

4 1 ∼ ∫

x

x tdt2

² =

x21 . ♦ F(x) = dt

tttx.

1ln.

1∫ + ∼ ∫x

dtt1

.ln ∼ x.ln x.

+ − + − + + x-1 ( )− + + 1 γ ( )ln x12

x112

x2 172

x3 1480

x4 ( )O x5

Page 28: Exercices intégrales généralisées

28

Exercice 6 : Nature de la série ∑+∞

=1nnu , où un = ∫

∞+

+++n tttt

dt2/12/32/52/7

. Equivalent du reste.

Solution :

La fonction intégrée f(t) est continue positive et intégrable sur [1, +∞[ car équivalente à 2/71

t.

un existe donc. En vertu du T.I.R.C., un ∼ ∫+∞

n tdt

2/7 = 52

2/51

n. Donc la série ∑

+∞

=1nnu converge.

Enfin, en vertu du T.S.R.C., Rn = ∑+∞

+= 1nkku ∼

52 ∑

+∞

+= 12/5

1nk k

.

Enfin, un encadrement intégral laissé au lecteur donne ∑+∞

+= 12/5

1nk k

∼ ∫+∞

n tdt

2/5 = 32

2/31

n.

Finalement, Rn = ∑+∞

+= 1nkku ∼

154

2/31

n.

Exercice 7 : Etudier la fonction F(x) = ∫∞+ −

x

t

dtt

e. .

Solution : 1) Généralités. La fonction f(t) = t

e t− est définie, continue et positive sur R*.

Elle est intégrable au V(+∞), car f(t) ≤ te− pour t ≥ 1

Elle est intégrable au V(±0), car f(t) ∼ t

1 au V((±0).

Par conséquent, le domaine de définition de F est R. De plus, il est clair que F est décroissante sur R. Sa limite en +∞ est nulle par définition d’un reste. Sa limite en −∞ est +∞, car f(t) → +∞ en −∞.

Notons que F(0) = ∫∞+ −

0.dt

t

e t

= Γ(½) = π .

2) Que dit Maple ? > with(plots):f:=t->exp(-t)/sqrt(abs(t)); > plot(f(t),t=-3..5,0..8,thickness=2);

Page 29: Exercices intégrales généralisées

29

> F:=x->int(f(t),t=x..infinity);F(x);F(0);

+ π

−I π ( )erf I −x ≤ x 0

− π ( )erf x < 0 x

> asympt(F(x),x,5);

− + − 1x

12

1x

( )/3 234

1x

( )/5 2158

1x

( )/7 2

eeeex

> series(F(x),x=0,4); Error, (in series/Heaviside) no series at 0

π > plot(F(x),x=-2..1,thickness=2,color=blue,numpoints=1000);

3) Retour à F.

Si x ≥ 0, F(x) = ∫∞+ −

x

t

dtt

e. = 2∫

+∞−

x

u due .² = π − 2∫ −x

u due0

². : fonction parfaitement connue.

Elle a en 0+ un développement limité selon les puissances de x , ce qui explique la tangente verticale en 0+. Elle a en +∞ un développement asymptotique à tous ordres obtenu par des intégrations par parties répétées.

Page 30: Exercices intégrales généralisées

30

Si x < 0, écrivons F(x) = ∫ −

−0.

x

t

dtt

e + π = π + 2 ∫

−xu due

0

². . Ici encore, fonction parfaitement

connue. Son étude en 0− permettrait d’expliquer sans peine la tangente verticale en 0. Son étude en −∞ s’obtient par des intégrations par parties laissées au lecteur.

Exercice 8 : 1) Nature et calcul de ∫ ∫+∞ +∞

−0

²..x

t dtedx .

2) Plus généralement, soit q : R+ → R+ une fonction continue, telle que x.q(x) soit intégrable.

Nature et calcul de : ∫ ∫+∞ +∞

0).(.

xdttqdx .

Solution :

1) Nature. La fonction f(t) = exp(−t2) est continue, positive et intégrable sur R, car 0 ≤ f(t) ≤

²11t+ ,

ou encore parce que 0 ≤ f(t) ≤ exp(−|t|) pour |t| ≥ 1.

La fonction F(x) = ∫+∞

−x

dtt²).exp( est elle-même continue, positive, et vérifie, pour tout x ≥ 1

0 ≤ F(x) ≤ ∫+∞

−x

dtt).exp( = exp(−x) ; F est donc intégrable.

Variante : Une intégration par parties donne un équivalent de F(x) en +∞ : F(x) ∼ x

x2

²)exp(−.

2) Calcul. Intégrons par parties :

∫A

dxxF0

).( = x.F(x) | A0 + ∫ −

Adxxx

0²).exp(. = A.F(A) − ²)exp(

21 x− | A

0

tend vers 21 quand A → +∞ , car 0 ≤ A.F(A) ≤ A.exp(−A) pour A ≥ 1.

3) Autre solution : intégrales doubles impropres de fonctions positives

∫ ∫+∞ +∞

−0

²..x

t dtedx = ∫∫ −D

t dtdxe .² où D = (x, t) ; 0 ≤ x ≤ t .

= ∫ ∫+∞

−0 0

²..t

t dxedt = ∫+∞

−0

²).exp(. dttt = − ²)exp(21 t− |+∞

0 = 21 .

4) La généralisation est facile, et laissée au lecteur.

Exercice 9 : Nature et calcul de ∫ ∫∞+ ∞+ −

0)..(

x

t

dxdtt

e.

Solution : La fonction t

e t− est intégrable sur toute demi-droite [x, +∞[, x > 0, mais pas sur ]0, 1].

La fonction F(x) = dtt

ex

t

.∫∞+ −

est définie, positive, et de classe C1 sur ]0, +∞[.

De plus, pour x ≥ 1, 0 ≤ F(x) ≤ dtex

t.∫+∞

− = exp(−x).

Une intégration par partie fournirait même un équivalent de F en +∞.

Si 0 < x ≤ 1, écrivons : F(x) = dtt

ex

t

.1

∫−

+ dtt

e t

.1∫

∞+ − = dt

tx.1

1

∫ + dtt

ex

t

.11

∫−−

+ dtt

e t

.1∫

∞+ −

Donc F(x) = − ln x + dtt

e t

.11

0∫−−

+ dtt

e t

.1∫

∞+ − + o(1) quand x → 0.

Finalement, F est intégrable sur ]0, +∞[. Pour calculer son intégrale, intégrons par parties :

∫A

dxxFε

).( = A.F(A) − ε.F(ε) − ∫A

dxxFxε

).('. = A.F(A) − ε.F(ε) + ∫ −A

x dxeε

.

Page 31: Exercices intégrales généralisées

31

= A.F(A) − exp(−A) + exp(ε) + ε.F(ε) → 1 quand ε → 0+ et A → +∞.

Conclusion : ∫ ∫∞+ ∞+ −

0)..(

x

t

dxdtt

e = 1.

5. Intégrales semi-convergentes.

La notion d’intégrale semi-convergente est au programme, mais celui-ci ne contient aucun théorème

les concernant. Il faut donc s’inspirer des méthodes utilisées lors de l’étude de ∫+∞

0.sin dt

tt .

Exercice 1 : Discuter la nature des intégrales ∫+∞

πdt

tt

a .sin et ∫+∞

0.sin dt

tt

a .

Solution : ∫+∞

πdt

tt

a .sin est • absolument convergente pour a > 1 ,

• semi-convergente pour 0 < a ≤ 1 , • divergente pour a ≤ 0 .

Le 1er point est évident. Le 2ème s’établit par intégration par parties.

Le 3ème s’établit en montrant que les tranches de Cauchy dtt

tn

n a .sin)1(

∫+ π

π ne tendent pas vers 0.

∫π

0.sin dt

tt

a converge ssi a < 2 (règle de l’équivalent). Au final, ∫+∞

0.sin dt

tt

a converge ssi 0 < a < 2.

Exercice 2 : Comparer les intégrales dttt.sin

1∫+∞

et dtt

ttt .]²sinsin[

1∫+∞

+ .

Solution : Les fonctions f(t) = ttsin et g(t) =

ttsin +

tt²sin =

ttsin +

t21 −

tt

2)2cos(

sont équivalentes en +∞. Cependant, f est semi-intégrable, g ne l’est pas, car elle est somme de deux fonctions semi-intégrables (par I.P.P.) et d’une fonction (celle du milieu) dont l’intégrale diverge. Cela montre que la règle de l’équivalent ne s’applique pas pour les fonctions semi-intégrables.

Ce contre-exemple est à rapprocher des deux séries : ∑+∞

=

−−1

1)1(

n

n

n et ∑

+∞

=

− +−1

2/3

1.)1(

n

n

nnn .

Exercice 3 : Étudier la nature des intégrales suivantes : ∫+∞

0).sin( dxex , ∫

+∞

∞−dxex).sin( ,

∫−1

0

]/1[

.)1(

dtt

t

(et calcul) , ∫+∞

+0.

cossin dt

ttt , ∫

+∞

0).1sin().sin( dt

tt , ∫

+∞+

0).1sin(

tdt

tt

∫+∞

+0

).1sin( dtt

t , ∫+∞

−1

).1sin().1tan( dttt

, ∫+∞

+0.

cos²cos dt

ttt

a (a > 0) , ∫

+∞

++0.

)1ln(cossin dt

ttt

Solution :

1) Le changement de variable u = ex donne ∫

Xx dxe

0).sin( = ∫

)exp(

1.sinXdu

uu .

Lorsque X tend vers +∞, ceci tend vers ∫+∞

1.sin du

uu . Il y a semi-intégrabilité.

2) Le même changement de variable donne ∫Y

X

x dxe ).sin( = ∫)exp(

)exp(.sinY

Xdu

uu .

Page 32: Exercices intégrales généralisées

32

Lorsque X tend vers −∞, et Y vers +∞ indépendamment, ceci tend vers ∫+∞

0.sin du

uu . Idem.

4) La fonction f(t) = tt

tcos

sin+

est définie et continue sur [0, +∞[.

Ecrivons f(t) = ttsin ( 1 −

ttcos + O(

t1) ) =

ttsin −

tt

2)2sin(

+ O( 2/31

t) .

f est somme de deux fonctions semi-intégrables et d’une fonction intégrable.

Donc l’intégrale ∫+∞

+0.

cossin dt

ttt converge.

5) La fonction f(t) = sin t. sint1 est continue sur ]0, +∞[.

Au V(0+), f est bornée, donc intégrable. Mieux même ! | f(t) | ≤ t ; il y a fausse impropreté.

Au V(+∞), écrivons : f(t) = t

tsin + O( 31t

) ; f est somme d’une fonction semi-intégrable et d’une

fonction intégrable. Conclusion : l’intégrale ∫+∞

0).1sin().sin( dt

tt converge.

6) La fonction f(t) = sin( t + t1)

t1 est continue sur ]0, +∞[.

Au V(0+), | f(t) | ≤ t

1 , donc f est intégrable sur ]0, 1].

Au V(+∞), écrivons f(t) = t

1 [ sin t.cost1 + sin

t1.cos t ] =

ttsin + O( 2/5

1t

) + O( 2/31

t).

f est somme d’une fonction semi-intégrable et de deux fonctions intégrables.

Conclusion : l’intégrale ∫+∞

+0

).1sin(t

dtt

t est convergente.

7) La fonction f(t) = sin( t + t

1 ) est continue sur ]0, +∞[.

Ecrivons f(t) = sin t.cost

1 + sint

1 .cos t.

Au V(0+), f(t) = o(1) + O(t

1 ) = O(t

1 ), donc f est intégrable sur ]0, 1].

Au V(+∞), f(t) = ttsin −

tt

2sin + O( 2/3

1t

) + ttcos + O( 2/3

1t

).

F est somme de trois fonctions semi-intégrables et de deux fonctions intégrables.

Conclusion : l’intégrale ∫+∞

+0

).1sin( dtt

t converge.

Exercice 4 : Nature de l’intégrale ∫+∞

+0.

1²ln.sin dx

xxx .

Solution : [ Mines 1993, RMS n° 219 ]

C’est l’intégrale impropre en 0+ et en +∞ de la fonction f(x) = 1²

ln.sin+x

xx .

Au V(0+), f(x) ∼ x.ln x → 0 : intégrale faussement impropre.

Au V(+∞), la majoration | f(x) | ≤ 1²

ln.+xx ne conclut pas.

Page 33: Exercices intégrales généralisées

33

Ecrivons 1²

1+x

= x1 + O( 3

1x

) , donc f(x) = xx

x ln.sin + O( 3ln.sin

xxx ) = x

xx ln.sin + O( 2

1x

).

g(x) = xx

x ln.sin est semi-intégrable sur [1, +∞[. En effet une intégration par parties donne :

dtt

ttX.ln.sin

1∫ = − t

tt ln.cos X1 + dt

ttt

X.

²ln1.cos

1∫− . Or 2

)ln1(cost

tt − = O( 2/3

1t

) …

Exercice 5 : Natures des intégrales : ∫+∞

−0

3 ).sin(. dxxxx et ∫+∞

0).(sin. dxxPx , P ∈ R[X] deg P ≥ 3.

Solution : [ Oral Centrale 1994, RMS n° 275 ].

La première intégrale est l’intégrale impropre en +∞ de la fonction continue f(x) = x.sin(x3 – x).

Cette fonction oscille fortement entre x et – x. Plusieurs approches sont possibles. Observons

d’abord que la fonction x → x3 – x est un C

∞-difféomorphisme croissant de [1, +∞[ sur [0, +∞[.

1ère approche : intégration par parties.

F(X) = ∫+ −X

dxxxx1

3 ).sin(. = ∫+ −−−X

xxdxx

1

3 ))cos((.1²3

= −1²3 −X

X cos(X3 – X) +

21 − ∫+ −−

+Xdxxx

xx

1

3 ).cos(.)²1²3(

1²3 .

Or −1²3 −X

X cos(X3 – X) → 0 quand X → +∞, et l’intégrale∫+ −−

+Xdxxx

xx

1

3 ).cos(.)²1²3(

1²3 est absolument

convergente. On en conclut que l’intégrale ∫+∞

−0

3 ).sin(. dxxxx converge.

Elle est d’ailleurs semi-convergente, car :

∫+ −X

dxxxx1

3 .)sin(. ≥ ∫+ −X

dxxxx1

3 ).²(sin. = ∫+−−X

dxxx

x1

3

.2

))(2cos(1.

= 4

1²−X − 21 ∫+ −

Xdxxxx

1

3 )).(2cos(. .

Or l’intégrale ∫+∞

+−

1

3 )).(2cos(. dxxxx converge pour les mêmes raisons que ci-dessus.

Donc ∫+ −X

dxxxx1

3 .)sin(. → +∞ quand X → +∞.

2ème approche : changement de variable.

Posons u = x3 – x. La bijection réciproque x = g(u) est implicite (à moins de recourir aux formules

de Cardan), mais enfin : ∫+∞

+−

1

3 ).sin(. dxxxx = ∫+∞

0).(').sin().( duuguug .

Or g(u).g’(u) = 1²3 −x

x , et x ∼ u1/3

, puis, après calculs, x = g(u) = u1/3

+ 31 u

−1/3 + O(u

−5/3).

On en déduit que g(u).g’(u) = 31 u

−1/3 + O(u

−5/3).

Ainsi, ∫+∞

1).(').sin().( duuguug =

31 ∫

+∞

+1 3/1 .sin duu

u + ∫+∞

+1 3/5 ).1( duu

O .

C’est la somme d’une intégrale semi-convergente et d’une intégrale absolument convergente. Ces deux méthodes se généralisent à la seconde intégrale.

Exercice 6 : Calcul de I = dtt

t.sin0∫+∞

.

1) a) Montrer que la fonction f(x) = x1 −

xsin1 est, une fois prolongée, de classe C1 sur ]−π, +π[.

Page 34: Exercices intégrales généralisées

34

b) En déduire que limn→+∞ ∫2/

0).sin().(

πdxnxxf = 0 .

2) Pour tout n ∈ N, on pose : Jn = ∫2/

0.

sin)sin(π

dxx

nx.

Trouver une relation entre Jn et Jn−2 ( n ≥ 2 ). En déduire Jn .

3) Déduire de ce qui précède les valeurs de I et J = dtt

t.²²sin

0∫+∞

.

Solution :

0) La fonction x

xsin est semi-intégrable, et non intégrable, sur R+ : question de cours.

1.a) La fonction f(x) = x1 −

xsin1 est, une fois prolongée, de classe C

1 sur ]−π, +π[.

Tout d’abord, elle est C∞

sur ]−π, 0[ ∪ ]0, +π[.

Le fait qu’elle soit de classe C1 sur ]−π, +π[ peut se montrer de deux façons.

1ère méthode : élémentaire. Elle consiste à faire un développement limité de f et de f’ en 0.

Il vient f(x) = −6x + O(x

2), ce qui montre que f(x) → 0 et f’(0) = −

61 .

Puis f’(x) = −²

1x

+ xx²sin

cos = −61 + O(x), ce qui montre que f’(x) → f’(0) = −

61 quand x → 0 .

2ème méthode : f est développable en série entière au V(0), donc C∞

en 0.

En effet, f(x) = xxxx

sin.sin − . Après simplification par x

2, on trouve f(x) =

)()(

xbxa

, où a et b sont sommes

de séries entières de rayon infini, et b(0) = 1. Le théorème de division des séries entières s’applique et montre que f est DSE(0).

b) Conséquence : limn →+∞ ∫2/

0).sin().(

πdxnxxf = 0 .

On reconnaît le lemme de Riemann-Lebesgue. Il repose sur la continuité de f en 0. Mais ici, on peut l’établir élémentairement, par intégration par parties et majoration en valeur absolue :

In = ∫ −2/

0))cos(().(

π

nnxdxf = − 2/

0)cos().( π

nnxxf

+ ∫2/

0).('.)cos(π

dxxfnnx

= O(n1 ) .

2) Soit Jn = ∫2/

0.

sin)sin(π

dxx

nx. Pour n ≥ 2, il vient :

Jn − Jn−2 = ∫−2/

0.

sinsin).)1cos((.2π

dxx

xxn = 2∫ −

2/

0).)1cos((

πdxxn =

12−n

sin((n−1)2π ) .

Jn − Jn−2 = 0 si n est impair : la suite (J2k+1) est constante égale à 2π .

Jn − Jn−2 = 2 12

)1( 1

−− +

k

k

si n = 2k. Donc J2k = 2 ( 1 − 31 +

51 – … +

12)1( 1

−− +

k

k

) .

3) Conclusion. Ecrivons

Jn = ∫2/

0.)sin(πdx

xnx

− ∫2/

0).sin().(

πdxnxxf = ∫

2/

0.sinπndt

tt − ∫

2/

0).sin().(

πdxnxxf .

On déduit de c) que (Jn) tend vers ∫+∞

0.sin dt

tt .

Faisons tendre n vers l’infini par valeurs impaires. Il vient ∫+∞

0.sin dt

tt =

2π .

Remarque : si l’on fait tendre n vers l’infini par valeurs paires, on trouve ∑+∞

= +−

0 12)1(

k

k

k =

4π .

Page 35: Exercices intégrales généralisées

35

Enfin, l’intégrale J = dtt

t.²²sin

0∫+∞

se calcule par parties :

J = ∫+∞

−0

)1(.²sint

dt = ∫+∞

0.cos.sin2tdttt = ∫

+∞

0.)2sin( dt

tt

= ∫+∞

0.)sin( du

uu

= 2π .

Exercice 7 : Bis repetita. Calculer successivement : Sn = ∑=

−n

k

xk1

))12sin(( , In = ∫2/

0.

²sin²sinπ

dxxnx ,

Jn = ∫2/

0.²cot.²sin

πdxxgnx , I = dx

xx.

²²sin

0∫+∞

et J = dxx

x.sin0∫+∞

.

Solution :

Exercice 8 : Convergence et calcul de dtt

btat.

)cos()cos(0∫+∞ −

, a et b > 0 (Intégrale de Frullani).

Solution : Notons I(a, b) cette intégrale.

Elle est faussement impropre en 0, car t

btat )cos()cos( − = O(t) au V(0+).

Elle est semi-convergente sur [1, +∞[, comme différence de deux intégrales semi-convergentes.

Pour calculer I(a, b), écrivons : I(a, b) = limε→0+ dtt

btat .)cos()cos(∫

+∞ −ε

.

dtt

btat .)cos()cos(∫

+∞ −ε

= dttat .)cos(

∫+∞

ε − dt

tbt .)cos(

∫+∞

ε = dx

xx

a.cos∫

+∞

ε − dx

xx

b.cos∫

+∞

ε

= dxx

xb

a.cos∫

ε

ε = dx

xb

a.1∫

ε

ε + dx

xxb

a.1cos∫ −ε

ε = ln

ab + dxxf

b

a.)(∫

ε

ε,

où f(x) = xx 1cos − est une fonction prolongeable par continuité en 0.

Par conséquent, dxxfb

a.)(∫

ε

ε → 0 quand ε → 0+. Au final :

dtt

btat .)cos()cos(0∫+∞ −

= lnab .

Remarque : c’est la même technique que le calcul de dttee btat

.0∫

∞+ −− − avec une légère différence.

Exercice 9 : Convergence et calcul de I(x, y) = dtt

ytxt .²

)sin()..sin(0∫+∞

.

Solution : [ Oral X 1984, RMS n° 126 ]

1) Intégrale faussement impropre en 0, absolument convergente sur [1, +∞[.

2) Ecrivons I(x, y) = 21 dt

ttyxtyx .

²))cos(())cos((

0∫+∞ +−−

= 21 [ K(x + y) – K(x – y) ] ,

où K(x) = dtt

xt .²

)cos(10∫+∞ −

= x duu

u.²

cos10∫+∞ − si x > 0.

Une intégration par parties donne duu

u.²

cos10∫+∞ − = du

uu.sin

0∫+∞

= 2π .

De plus K(0) = 0. Par parité, K(x) = 2π |x|.

Conclusion : I(x, y) = dtt

ytxt .²

)sin()..sin(0∫+∞

= 4π [ | x + y | − | x – y | ].

Page 36: Exercices intégrales généralisées

36

Exercice 10 : Convergence et calcul de A = ∫∞+

0 2

3

.sin

dxx

x et de B = ∫

∞+

0 3

3

.sin

dxx

x.

Solution : [ Oral X 1993, RMS n° 79 ]

Les deux intégrales sont faussement impropres en 0, et absolument convergentes sur [1, +∞[.

Ecrivons A = ∫+∞ −0 2 .

4)3sin(sin.3 dx

xxx

= limε→0+ ∫+∞ −

εdx

xxx .

4)3sin(sin.3

2

= limε→0+ ∫+∞

εdx

xx.

4sin.3

2 − ∫+∞

εdx

xx .

4)3sin(

2 = limε→0+ 43 F(ε) −

43 F(3ε) , où F(ε) = ∫

+∞

εdx

xx.sin

2 .

Pour obtenir cette limite, étudions F au voisinage de 0+. Une IPP donne :

F(ε) = εεsin + ∫

+∞

εdx

xx.cos = ε

εsin + ∫1

.cosε

dxx

x + ∫+∞

1.cos dx

xx

= εεsin + ∫

1

ε xdx + ∫ −1

.1cosε

dxxx + ∫

+∞

1.cos dx

xx = − ln ε + C + o(1),

où C = 1 + ∫ −1

0.1cos dx

xx + ∫

+∞

1.cos dx

xx . On en déduit aussitôt :

A = ∫∞+

0 2

3

.sin

dxx

x =

43 ln 3.

Variante par Frullani :

43 F(ε) −

43 F(3ε) =

43

εεsin − ε

ε4

)3sin( −

43 ∫

+∞ −ε

dxx

xx .cos)3cos( →

43 ∫

+∞ −0

.)3cos(cos dxx

xx...

Les mêmes techniques permettent de montrer que B = ∫∞+

0 3

3

.sin

dxx

x =

83π .

On peut calculer plus généralement les intégrales I(n, p) = ∫∞+

0.

sindx

xx

n

p

lorsqu’elles convergent.

Remarque : Pour une raison que j’ignore, Maple se trompe dans le calcul de A. > F:=(n,p,x)->int(sin(t)^p/t^n,t=x..infinity);J:=(n,p)->int(sin(t)^p/t^n,t=0..infinity); F(3,2,x);J(3,2);F(3,3,x);J(3,3);

Exercice 11 : Montrer qu’au voisinage de + ∞ :

∫+∞

xdt

tt .²sin

α ∼ 1)1(21

−− αα x si α > 1 , ∫

+∞

xdt

tt.²sin ∼

21 lnx , ∫

+∞

xdt

tt .²sin

α ∼ )1(2

1

αα

−−x

si 0 < α < 1.

Solution :

Exercice 12 : Nature et calcul des intégrales ∫+∞

0( ∫

+∞

xdt

tt.sin ).dx et ∫

+∞

0( ∫

+∞

xdt

tt.cos ).dx.

Solution :

Rappelons pour commencer que x

xsin est semi-intégrable sur R+, et x

xcos semi-intégrable sur toute

demi-droite [x, +∞[, x > 0, mais non intégrable sur ]0, 1].

−14

− + − + 1 ( )cos 2x 2 ( )sin 2x x 4 ( )Ci 2 x x2

x2

−18

− + + + − − − 3 π x2 ( )sin 3x 3 ( )cos 3x x 9 ( )Si 3x x2 3 ( )sin x 3 ( )cos x x 3 ( )Si x x2

x2

38

π

Page 37: Exercices intégrales généralisées

37

1) La fonction F(x) = ∫+∞

xdt

tt.sin est définie et de classe C

1 sur R+. Il faut faire deux intégrations

par partie pour obtenir un renseignement sur son comportement asymptotique en +∞.

F(x) = ∫+∞

xdt

tt.sin =

xxcos +

²sinx

x − 2 ∫+∞

xdt

tt.sin

3 = x

xcos + O(²

1x

) (*).

Du coup, F est somme d’une fonction semi-intégrable et d’une fonction intégrable sur [1, +∞[.

Comme F a une limite en 0, l’intégrale ∫+∞

0).( dxxF est faussement impropre en 0 et semi-conver-

gente en +∞. Pour la calculer, intégrons par parties derechef :

∫A

dxxFε

).( = A.F(A) − ε.F(ε) − ∫A

dxxFxε

).('. = A.F(A) − ε.F(ε) + ∫A

dxxε

.sin

= A.F(A) − ε.F(ε) − cos A + cos ε → 1 quand A → +∞ et ε → 0+, en vertu de (*).

2) La fonction G(x) = ∫+∞

xdt

tt.cos est définie et de classe C

1 sur R+*. Il faut faire deux intégrations

par partie pour obtenir un renseignement sur son comportement asymptotique en +∞.

G(x) = ∫+∞

xdt

tt.cos = −

xxsin +

²cos

xx − 2 ∫

+∞

xdt

tt.cos

3 = − x

xsin + O(²

1x

) (*).

Du coup, G est somme d’une fonction semi-intégrable et d’une fonction intégrable sur [1, +∞[. Attention, sur ]0, 1], la situation est moins simple qu’en 1). Ecrivons :

G(x) = ∫1

x tdt + ∫ −1

.1cosx

dttt + ∫

+∞

1.cos dt

tt = − ln x + ∫ −1

0.1cos dt

tt + ∫

+∞

1.cos dt

tt + o(1).

Donc G(x), équivalente à − ln x, est intégrable sur ]0, 1]. Intégrons par parties derechef :

∫A

dxxGε

).( = A.G(A) − ε.G(ε) − ∫A

dxxGxε

).('. = A.G(A) − ε.G(ε) + ∫A

dxxε

.cos

= A.G(A) + sin A − ε.G(ε) − sin ε → 0 quand A → +∞ et ε → 0+.

Conclusion : ∫+∞

0( ∫

+∞

xdt

tt.sin ).dx = 1 et ∫

+∞

0( ∫

+∞

xdt

tt.cos ).dx = 0.

Avec Maple, qui reste perplexe (et se trompe) : > F:=x->int(sin(t)/t,t=x..infinity);F(x);series(F(x),x=0,4); asympt(F(x),x,4);int(F(x),x=0..infinity);

> G:=x->int(cos(t)/t,t=x..infinity);G(x);series(G(x),x=0,4); asympt(G(x),x,4);int(G(x),x=0..infinity);

Exercice 13 : 1) On considère la fonction F(x) = ∫x

dtt0.1sin .

a) Domaine de définition de F. Dérivabilité de F sur R*, en 0 ? Equivalent de F en +∞ ? b) Variations et graphe de F ?

:= F → x d⌠

x

( )sin tt

t − 12

π ( )Si x

− + + 12

π x118

x3 ( )O x4

+ − + ( )cos xx

( )sin x

x2

2 ( )cos x

x3

O

1

x4

0

:= G → x d⌠

x

( )cos tt

t − ( )Ci x + + ( )− − γ ( )ln x14

x2 ( )O x4

− + + + ( )sin xx

( )cos x

x2

2 ( )sin x

x3

O

1

x4lim → x ∞

− + x ( )Ci x ( )sin x

Page 38: Exercices intégrales généralisées

38

2) Soit f : R → R une fonction continue T-périodique (T > 0) ; on pose ω = T1 ∫

Tdttf

0).( .

a) Domaine de définition de F(x) = ∫x

dtt

f0

).1( .

b) Dérivabilité de F sur R*, en 0 ? (Oraux ENS, 2002)

Solution :

Exercice 14 : Nature de l’intégrale ∫+∞

0).sin(sin dtt ?

Solution : La fonction f(t) = sin(sin t) est continue 2π-périodique. Posons-nous plus généralement le problème : si f est continue 2π-périodique, à quelle condition

∫+∞

0).( dttf converge ? La réponse est : elle converge ssi f est nulle.

Exercice 15 : Soit f : [0, +∞[ → R uniformément continue. Montrer que l’intégrale ∫+∞

0

)( .dte tif

diverge.

Solution :

6. Espaces fonctionnels.

Exercice 1. On pose L1(I) = f ∈ C(I, R) ; f intégrable et L

2(I) = f ∈ C(I, R) ; f

2 intégrable

pour tout intervalle I de R. Etudier les relations d’inclusion entre L1(I) et L

2(I) dans chacun des cas

suivants : I = [0, 1], I = ]0, 1], I = [0, +∞[.

Solution : [ Oral Mines MP 2010, RMS n° 481 ]

♦ Si I = [0, 1], il n’y a pas de problème L1(I) = L

2(I) = C(I, R).

♦ Si I = ]0, 1], C(I, R) ⊃ L1(I) ⊃ L

2(I), les inclusions étant strictes.

En effet de l’inégalité 2.| f | ≤ f 2 + 1, on déduit que si f est de carré intégrable, f est intégrable.

Car la fonction 1 est intégrable sur I.

Les inclusions sont strictes, car ax1 est toujours élément de C(I, R) ; elle est élément de L

1(I) ssi a <

1, de L2(I) ssi a < ½.

♦ Si I = [1, +∞[, C(I, R) contient strictement L1(I) et L

2(I), mais il n’y a aucune inclusion entre ces

deux espaces.

Les inclusions sont strictes, car ax1 est toujours élément de C(I, R) ; elle est élément de L

1(I) ssi a >

1, de L2(I) ssi a > ½. Donc

x1 est élément de L

2(I) et non de L

1(I).

Reste à trouver une fonction f intégrable sur I, non de carré intégrable…

Ce n’est pas si facile, car si f tend vers 0 en l’infini, f 2 ≤ | f | , donc f intégrable implique f de carré

intégrable… Il faut donc chercher une fonction ne tendant pas vers 0 en l’infini…

Considérons d’abord la fonction f telle que f(x) = n pour n − 31n

≤ x ≤ n + 31n

, pour tout n ≥ 1, nulle

ailleurs. Cette fonction est intégrable, mais non de carré intégrable. Si l’on veut une fonction

Page 39: Exercices intégrales généralisées

39

continue, considérer la fonction continue affine par morceaux g telle que g(n) = n, g(n ± 31n

) = 0,

g étant affine ailleurs.

Exercice 2 : Soit f ∈ C1(R+, R) telle que les intégrales ∫+∞

0).²(². dttft et ∫

+∞

0).'²( dttf convergent.

Montrer que ∫+∞

0).²( dttf converge, et que : ∫

+∞

0).²( dttf ≤ 2. ∫

+∞

0).²(². dttft . ∫

+∞

0).'²( dttf .

avec égalité ssi f est de la forme f(t) = A.exp(−λ.t2) (A ∈ R, λ > 0).

Solution : Inégalité de Weyl. Cf. mes problèmes d’intégration.

Exercice 3 : Soit f ∈ C2(R+, R) telle que f et f '' soient de carré intégrable.

1) Montrer que f ' est de carré intégrable, puis que f.f' est intégrable. 2) Démontrer que limt→+∞ f(t) = 0 ; en est-il de même de f '(t) ?

3) Démontrer enfin que ∫+∞

0).'²( dttf ≤ ∫

+∞

0).²( dttf + ∫

+∞

0).''²( dttf . Cas d’égalité ?

[ Indication : on pourra considérer F(A) = ∫A

0f 2 − f '2 + f ''2 − ( f + f ' + f '' )2 ]

Solution : Inégalité de Kolmogorov. Cf. mes problèmes d’intégration.

Exercice 4 : Soit f ∈ C2(R, R) telle que f et f'' soient de carré intégrable sur R.

Montrer que f' est de carré intégrable, et que ∫R f'2 ≤ ( ∫R f2 ).( ∫R f''2 ). Cas d’égalité ? Solution :

7. Convergence monotone, convergence dominée.

Exercice 1 : Soit f : R → R continue à valeurs > 0. Pour tout n ∈ N*, soit an = ∫∞+

∞− +

²1)(

)²²exp(

nxf

nx

dx.

Montrer l’existence de an. Trouver une cns pour que la suite (an) converge.

Solution :

Exercice 2 : Soit (an) une suite de réels > 0 tendant vers 0. Pour tout x > 0, on pose

N(x) = card n ∈ N ; an ≥ x . Montrer que la fonction N est définie et monotone ; limites en 0+ et + ∞ ?

Montrer que ∑+∞

=0n

na converge ssi N est intégrable sur ]0, +∞[, et qu’alors ∑+∞

=0n

na = ∫+∞

0).( dxxN .

Solution : [Oral Mines PSI 2011, RMS n° 602 ]

Exercice 3 : calcul de l’intégrale de Gauss ∫R ²xe− dx = π .

a) Montrer que ²xe− est intégrable sur R.

Page 40: Exercices intégrales généralisées

40

On rappelle l’équivalent de Wallis Wn = ∫2/

0.sin

πdttn ∼

n2π .

b) Montrer que ∫R ²xe− dx = limn→+∞ ∫R n

nx

dx)²1( +

; en déduire cette valeur.

c) Montrer que ∫R ²xe− dx = limn→+∞ ∫−n

n(1 −

nx² )n.dx ; en déduire cette valeur.

Solution :

a) h(x) = ²xe− est continue positive et intégrable car h(x) ≤ ²1

1x+ , ou encore car h(x) ≤ xe− si |x| ≥ 1.

b) Considérons la suite de fonctions fn(x) = n

nx )²1(1

+.

Ces fonctions sont toutes intégrables et convergent simplement vers la fonction h(x) = ²xe− .

De plus 0 ≤ fn(x) ≤ f1(x). Le TCD s’applique.

On peut même montrer que fn(x) tend en décroissant vers h(x).

Le changement de variable x = n tan ϕ, ou plutôt ϕ = Arctannx ramène à des Wallis :

Jn = ∫R n

nx

dx)²1( +

= 2 n ∫ −2/

0

)1(2 .cosπ

ϕϕ dn ∼ 2 n)22(2 −n

π ∼ π .

c) Considérons la suite de fonctions gn(x) = ( 1 −nx² )n

pour |x| ≤ n , gn(x) = 0 sinon.

Ces fonctions tendent simplement vers h(x). De plus 0 ≤ gn(x) ≤ h(x), qui est intégrable.

Le changement de variable x = n sin ϕ, ou plutôt ϕ = Arcsinnx ramène aussi à des Wallis.

On peut montrer que gn(x) tend en croissant vers h(x).

d) La remarque qui tue… L’encadrement gn(x) ≤ h(x) ≤ fn(x) permet de conclure via les gendarmes, sans utiliser le moindre théorème.

Exercice 4 : Limite de la suite In = ∫ −+n

xn dxenx

0

2 ..)1( .

Solution : [ Oral X 1982 ]

Ecrivons In = ∫+∞

0).( dxxfn , où fn(x) = ( 1 +

nx )

n.e

−2x pour 0 ≤ x ≤ n, 0 pour x > n.

fn(x) → e−x

pour tout x, et 0 ≤ fn(x) ≤ e−x

.

On conclut par convergence dominée que In → ∫+∞

−0

.dxe x = 1.

Page 41: Exercices intégrales généralisées

41

Remarque : en 1982, les programmes de taupe ne contenaient pas le théorème de convergence dominée. Il fallait démontrer ce résultat à la main, en cassant en morceaux. A noter que l’on peut

calculer In, soit par binôme, soit par parties.

Exercice 5 : Soit f : R → R continue et bornée. Existence et limite de In = πn ∫

+∞

∞−−− dxexf axn .)( )²( .

Solution : Il est facile de montrer que In existe pour n ≥ 1.

Il y a concentration d’aire au V(a), et l’on peut penser que la suite (In) se comporte comme

πn ∫

+∞

∞−−− dxeaf axn .)( )²( = f(a). π

n ∫+∞

∞−−− dxe axn .)²( = f(a), autrement dit que (In) tend vers f(a).

Cela peut se démontrer de plusieurs façons.

La plus simple est de faire le changement de variable u = (x – a) n .

Il vient : In = π1 ∫

+∞

∞−−+ due

nuaf u .)( ² .

Or fn(u) = f(a +n

u ).exp(−u2) → f(a).exp(−u

2) et | fn(u) | ≤ || f ||∞. exp(−u

2).

On conclut par convergence dominée.

Exercice 6 : Limites et équivalents de Jn = ∫+∞ −

0.dxe

nx et In = ∫

+∞

+

1.dxe

nx.

Solution : 1) Limites. Soit fn(x) = exp(−xn).

fn(x) est continue, positive, et intégrable sur [1, +∞[, car fn(x) ≤ f1(x) = exp(−x).

fn(x) → 1 si 0 ≤ x ≤ 1 , exp(−1) si x = 1 , 0 si x > 1. De plus, il y a une majorante intégrable :

Sur [1, +∞[, on peut écrire : 0 ≤ fn(x) ≤ f1(x) = exp(−x).

Sur [0, +∞[, on peut écrire : 0 ≤ fn(x) ≤ ϕ(x), où ϕ(x) = 1 sur [0, 1[, exp(−x) sur ]1, ∞[.

Le théorème de convergence dominée montre que Jn → 1, et que In → 0 (d’ailleurs en décroissant).

2) Equivalents : Le changement de variable y = xn donne resp. :

Jn = n1 ∫

+∞ −−0

11

. dyye ny et In = n1 ∫

+∞

+

−−1

11

. dyye ny .

Or, par I.P.P. : Jn = ∫+∞ −0

1

. dyye ny → ∫+∞ −0

dye y = 1 par convergence dominée.

En effet : gn(y) = nyye1

− → ye− simplement, et il y a domination par 0 ≤ gn(y) ≤ φ(y), où

φ(y) = ye− sur [0, 1] , ye y− sur [1, +∞[.

Et In = ∫+∞

+

−−1

11

. dyye ny → ∫+∞

+−−

1

1. dyye y derechef par convergence dominée.

En effet : gn(y) = 11−− nyye → 1−− ye y simplement, et il y a domination par 0 ≤ gn(y) ≤ ye− .

Conclusion : Jn → 1, In ∼ n1 ∫

+∞

+−−

1

1. dyye y .

Remarque : Jn = Γ(1 + n1 ) a un développement asymptotique à tous ordres, taylorien.

Jn − In = ∫−−1

0

11

. dyye ny est une fonction gamma incomplète, qui a aussi un d.a. à tous ordres.

Page 42: Exercices intégrales généralisées

42

Exercice 6 : Montrer que la suite In = ∫+∞

+++0 )1)...(2

1)(1(nxxx

dx est définie pour n ≥ 2. Limite ?

Solution : La suite de fonctions fn(x) = )1)...(

21)(1(

1

nxxx +++

est décroissante, et tend simplement

vers la fonction f(0) = 1 , f(x) = 0 pour x > 0 (f étant discontinue, il n’y a pas convergence uniforme.)

En effet, si x > 0, ln fn(x) = −∑=

+n

k kx

1

)1ln( , somme partielle d’une série divergente à termes positifs,

donc ln fn(x)→ −∞. Variante : − x Hn = −∑=

n

k kx

1

≤ ln fn(x) ≤ −∑=

−n

k kx

kx

1

)²2²( = − x Hn +

2²x Sn.

Or la suite (Hn) tend vers +∞, tandis que (Sn) est convergente, donc bornée.

Comme 0 ≤ fn(x) ≤ f2(x) sur R+, la suite (In) tend vers 0 en vertu du théorème de convergence dominée sur les intervalles quelconques. Il s’agit de la convergence dominée du pauvre, puisqu’il y a convergence uniforme sur tout segment inclus dans ]0, 1] ou ]0, +∞[.

Remarque : on peut obtenir un équivalent de In par cassage en 3 (cf. un problème d’intégration).

Exercice 7 : Pour tout n, soit un = ∫ +1

0.

1dt

tt

n

n

. a) Nature de la série ∑+∞

=−

0

.)1(n

nn u ?

b) Montrer que un = n2ln −

²12²n

π + o(²

1n

).

Solution : [ Oral Mines MP 2010, RMS n° 498 ]

Exercice 8 : Pour tout n, soit In = ∫+∞

+ +1 1 ntdt . 1) Limite L de la suite (In), puis équivalent de In – L.

2) Donner un développement asymptotique à 3 termes de In .

Solution :

1) Il est immédiat que In existe pour n ≥ 2, et que la suite (In) est décroissante.

fn(x) = nx+11 → ½ si x = 1, 0 si x > 1. De plus, 0 ≤ fn(x) ≤ 21

1x+ .

En vertu du théorème de convergence dominée, In → 0.

2) Le changement de variable u = tn donne In =

n1 du

uuu n

.)1(1

/1

∫∞+

+ + .

Jn = duuu

u n

.)1(1

/1

∫∞+

+ + → ∫+∞

+ +1 )1( uudu = ln 2 par convergence dominée.

En effet )1(

/1

uuu n

+ → )1(

1uu + et 0 <

)1(

/1

uuu n

+ ≤ )1(

1uu +

pour n ≥ 2. Donc In ∼ n2ln .

3) Pour avoir le terme suivant, écrivons :

n.( Jn – ln 2 ) = duuu

un n

.)1()1(

1

/1

∫∞+

+ +−

→ duuu

u .)1(

ln1∫+∞

+ + = dsss.

1ln1

0∫ +−

= dssss ....).²1.(ln1

0∫ ++−− = ∑+∞

=

−−1

1

²)1(

k

k

k =

12²π .

Page 43: Exercices intégrales généralisées

43

Justification laissée au lecteur par convergence dominée et intégration terme à terme des séries.

Conclusion : In = ∫+∞

+ +1 1 ntdt =

n2ln +

12²π

²1n

+ o(²

1n

) quand n → +∞.

Remarque : développement asymptotique à tous ordres.

In = n1 du

uue nu

.)1(1

/ln

∫∞+

+ + = n1 du

uunu

k k

k

k

.)1(.

ln.!

11 0∫ ∑

∞+

+

+∞

= + = ∑+∞

=0 !1

k k. 1

1+kn

duuuuk

.)1(

ln1∫∞+

+ + .

Il s’agit d’un développement en série entière en 1/n qui fournit des développement asymptotiques à tous ordres.

Exercice 9 : Limite et développement à deux termes de In = ∫+∞

+0 1 nxdx .

Solution : 1) Existence de la suite. Il est immédiat que In existe pour n ≥ 2.

2) Limite de la suite. fn(x) = nx+11 → 1 si 0 ≤ x < 1, ½ si x = 1, 0 si x > 1.

De plus, 0 ≤ fn(x) ≤ ϕ(x), où ϕ(x) = 1 si 0 ≤ x < 1, ½ si x = 1, 211x+ si x > 1.

En vertu du théorème de convergence dominée, In → 1. 3) Développement à deux termes.

4) Remarque finale. In est en fait une intégrale eulérienne, qui se rattache aux fonctions ∆, Β et Γ via un changement de variable, la formule d’Euler et celle des compléments (cf. mon chapitre sur les

intégrales eulériennes, § 7). Le changement de variable u = tn et ces formules donne en effet :

In = )1,1(.1nn

∆ = )1,11(.1nn

Bn

− = )1().11(.1nnn

Γ−Γ =

nn π

πsin.

.

On peut alors obtenir un développement asymptotique à tous ordres. > J:=n->Pi/(n*sin(Pi/n));asympt(J(n),n,15);

1

16

π2

n2

7360

π4

n4

3115120

π6

n6

127604800

π8

n8

733421440

π10

n10

1414477653837184000

π12

n12 + + + + + +

819137362124800

π14

n14

O

1

n16 + +

Exercice 10 : Montrer que : γ = 1 − ∫+∞

+

−1

][ dtt

tt = 1 − ∫ −

1

0]).1[1( dt

tt. En déduire 0 < γ < 1.

Solution : Montrons la convergence de chacune des deux intégrales. Les deux fonctions sont réglées sur tout

segment, et 0 ≤ f(t) = ²

][t

tt− ≤

²1t

et 0 ≤ g(t) = t1− [

t1] ≤ 1.

Le changement de variable u = 1/t ramenant une intégrale à l’autre, il suffit de montrer une formule.

Par exemple ∫+

−n

n

dttt

1

11

]).1[1( = ln( 1 + n1 ) −

11+n

, de sorte que :

∫+

−1

11

]).1[1(N

dttt

= ln(N + 1) − 21 −

31 − … −

11+N

→ 1 − γ quand N → +∞.

Page 44: Exercices intégrales généralisées

44

On en déduit 0 < γ < 1, car chacune des intégrales ∫+

−n

n

dttt

1

11

]).1[1( est > 0 et < )1(

1+nn

.

Exercice 11 : Montrer que ∀n ≥ 1 ∫ −n

n dxnx

x1.)1(1 − ∫ −−

1

0].)1(1[1 dx

nx

xn = ln n − ∑

=

n

k k1

1 .

En déduire γ = dxxe x

.11

0∫−−

− ∫∞+ −

1.dx

xe x

.

Solution : Notons I(n) le premier membre.

I(n) = ∫ −n

n dxnx

x1.)1(1 − ∫ −−

1

0].)1(1[1 dx

nx

xn = ∫

n

xdx

1 + ∫ −−

nn dx

nx

x1].1)1[(1 − ∫ −−

1

0].)1(1[1 dx

nx

xn

= ln n − ∫ −−n

n dxnx

x0].)1(1[1 = ln n − ∫ −−

1

0].)1(1[1 duu

un = ln n − ∫ −

−1

0.

11

dvvvn

= ln n − ∫ −+++1

0

1)....1( dvvv n = ln n − ∑=

n

k k1

1 .

Il reste à conclure par convergence dominée.

Exercice 12 : Montrer que ∫+∞

−− −−0

).11

1( dxxe

e xx = γ , où γ = limn ∑

=

n

k k1

1 − ln n .

Solution :

1) Convergence. La fonction f(x) = e−x ( xe−−1

1 − x1 ) est continue sur ]0, +∞[.

Elle est prolongeable par continuité en 0, car limx→0 f(x) = 21 ; intégrale faussement impropre en 0.

f(x) ∼ e−x en +∞ ; donc f est intégrable sur [1, +∞[, et finalement sur ]0, +∞[. Soit I sont intégrale.

2) Calcul. L’idée est d’utiliser la formule u−1

1 = 1 + u + u2 + … + u

n−2 +

uun

−−

1

1

.

Ecrivons I = ∫∞+

−−−−−− −−++++

0

)1()2( ).1

1...1( dx

xee

eee x

xnxnxx =

= ∫+∞ −−−− +++0

)2( )....1( dxeee xnxx + ∫+∞ −− −0

.dxx

ee xnx

+ ∫+∞

−− −−0

).11

1( dxxe

e xnx .

La première intégrale vaut 1 + 21 + … +

11−n

.

La deuxième est une intégrale de Frullani ; elle vaut – ln n (cf. exercice antérieur).

La troisième s’écrit Kn = ∫+∞

−0

).(. dxxbe nx , où b(x) = xe−−11 −

x1 est bornée sur ]0, +∞[, car elle tend

vers ½ en 0 et 0 en +∞. Or Kn tend vers 0,

soit en vertu du théorème de convergence dominée : e−nx

.b(x) → 0 et | e−nx

.b(x) | ≤ e−x

.| b(x) |,

soit par majoration élémentaire : | Kn | ≤ ∫+∞

−0

.)(. dxxbe nx ≤ B ∫+∞

−0

dxe nx = nB .

Références : Polya-Szegö, Problems and theorems in analysis, t. 1, n° 32 p. 54 Dieudonné, Calcul infinitésimal, n° 13, p. 163 (qui propose une autre solution) Tuloup, Cours d’analyse Centrale 1976 RMS 491 p. 206, etc.

Page 45: Exercices intégrales généralisées

45

Exercice 13 : Montrer que ∫+∞

−0

.ln dxxe x = − γ , où γ = limn ∑=

n

k k1

1 − ln n .

En déduire ∫+ −−1

0.

1dx

xe x

− ∫∞+ −

1.dx

xe x

= γ .

Solution :

1) Convergence. La fonction f(x) = e−x

.ln x est continue sur ]0, +∞[.

Au V(0+), f(x) ∼ ln x, qui est intégrable sur ]0, 1].

Au V(+∞), 0 ≤ f(x) ≤ e−x/2

ou 1/x² selon les goûts.

Conclusion : f est intégrable sur ]0, +∞[. Soit I sont intgrale.

Remarque : I = Γ’(1), car la fonction Γ(x) = ∫+∞

−−0

1 .dtet tx , définie sur ]0, +∞[, est de classe C1 et a

pour dérivée Γ’(x) = ∫+∞

−−0

1 ..ln dtett tx .

2) Calcul. Une idée est de monter que I = limn→+∞ ∫ −n

n dxxnx

0.ln.)1( .

Cela se montre par convergence dominée. Ecrivons : In = ∫ −n

n dxxnx

0.ln.)1( = ∫

+∞

0).( dxxfn ,

où fn(x) = (1 − nx )

n.ln x pour 0 < x ≤ n , 0 pour x ≥ n.

Or fn(x) → exp(−x).ln x et vérifie | fn(x) | ≤ exp(−x).| ln x |.

Reste à calculer l’intégrale In, ce qui n’est pas si facile ! Tout d’abord, par chgt de variable :

In = ∫ −1

0).ln(.)1( ndyyny n = n.ln n. ∫ −

1

0.)1( dyy n + n.∫ −

1

0).ln(.)1( dyyy n

= 1+n

n .ln n + n.∫ +−− +1

0

1

)1)1(1

().ln(n

ydy

n

Il y a nécessité d’introduire la constante 1

1+n

dans la forme différentielle sans quoi il y a problème !

Jn = ∫ +−− +1

0

1

)1)1(1

().ln(n

ydy

n

= − 1

1+n ∫

+−−1

0

1

.)1(1

dyyy n

( la partie intégrée étant nulle ).

= − 1

1+n ∫ −

− +1

0

1

.1

1dx

xxn

= − 1

1+n ∫ ++++

1

0)....²1( dxxxx n = −

11+n

Hn+1 .

En conclusion, In = 1+n

n [ ln n − Hn+1 ] , et l’on conclut aussitôt.

3) Conséquence. Intégrons par parties, mais attention ! il faut casser en deux.

∫+∞

−0

.ln dxxe x = ∫+

−1

0.ln dxxe x + ∫

+∞−

1.ln dxxe x = ∫

+−−

1

0)1(.ln xedx + ∫

+∞−−

1)(.ln xedx

= − ∫+ −−1

0.

1dx

xe x

+ ∫∞+ −

1.dx

xe x

. Voilà qui nous ramène à un exercice antérieur !

4) L’expression intégrale obtenue permet un calcul approché de la constante d’Euler.

Exercice 14 : Étudier les suites de fonctions suivantes, et leurs intégrales : fn(x) = na.x.( 1 − x )n sur [0, 1] et gn(x) = na.x.exp(−n.x) sur R+ . Le théorème de convergence dominée s’applique-t-il ?

Solution :

Page 46: Exercices intégrales généralisées

46

Exercice 15 : On considère la suite de fonctions fn : x ∈ [0, 1] → sin(nx) . Montrer qu’aucune suite extraite (fn(k)) ne converge simplement sur [0, 1] vers 0.

[ Indication : On pourra considérer la suite ∫[0,1] fn(k)(x)2.dx . ]

Solution : ∫1

0).²(sin dxnx =

21 −

nn

4)2sin(

→ 21 .

Si une suite extraite (fn(k)) convergeait simplement sur [0, 1] vers 0, la suite ∫1

0).²(sin dxxnk tendrait

vers 0 par convergence dominée. Or on vient de montrer le contraire.

Exercice 16 : Soit I un intervalle de R, (fn) une suite de fonctions continues par morceaux de I dans R, convergeant simplement vers f continue par morceaux.

1) On supppose (∀n) fn ≥ 0. Montrer limn→+∞ ∫I | fn − f | = 0 ⇔ limn→+∞ ∫I fn = ∫I f. 2) Contre-exemple si on enlève l’hypothèse (∀n) fn ≥ 0 ?

Solution :

8. Intégration terme à terme des séries.

Exercice 1 : Montrer que : ∫1

0.dxxx = ∑

+∞

=

−−1

1)1(

nn

n

n et que ∫ −

1

0.dxx x = ∑

+∞

=1

1n

nn.

Calculer ces intégrales à 10−6 près.

Solution : Il s’agit des deux « series mirabili » découvertes par Johann Bernoulli en 1697. Nous traiterons seulement la première intégrale.

1) Formellement ∫1

0.dxxx = ∫

1

0).ln.exp( dxxx = ∫∑

+∞

=

1

0 0

.!

lndx

nxx

n

nn

= ∑+∞

=0 !1

n n ∫1

0.ln. dxxx nn .

Reste à calculer les intégrales In = ∫1

0.ln. dxxx nn . Une première tentative d’intégration par parties

échoue, car elle ne ramène pas In à In−1 , mais In à ∫ −1

0

1 .ln. dxxx nn .

Il faut donc généraliser la question en considérant In(a) = ∫1

0.ln. dxxx na , pour a > 0.

Alors une ipp donne In(a) = 1+

−a

n In−1(a) ; donc In(a) = 1+

−a

n1

)1(+−−

an

…11+−a

I0(a) = 1)1(!)1(

++−

n

n

an

.

La situation rencontrée obéit au principe “ more may be less ”. On la rencontre aussi lors du calcul

de In = ∫ −1

0.)1.( dxxx nn : il est plus facile de calculer In,p = ∫ −

1

0.)1.( dxxx pn .

Revenons z-a nos moutons ! Finalement, ∫1

0.ln. dxxx nn = In(n) = 1)1(

!)1(++

−n

n

nn

, puis :

∫1

0.dxxx = ∑

+∞

=0 !1

n n 1)1(!)1(

++−

n

n

nn

= ∑+∞

=++

−0

1)1()1(

nn

n

n = ∑

+∞

=

−−1

1)1(

nn

n

n .

2) Reste à justifier l’interversion des limites. Ici, on a l’embarras du choix :

1ère méthode : la fonction f(x) = exp(x.ln x) est continue sur [0, 1] après prolongement par 1 en 0.

Elle est somme de la série ∑+∞

=0 !)ln(

n

n

nxx

.

Page 47: Exercices intégrales généralisées

47

Cette série est normalement convergente sur [0, 1], car la fonction, x.ln x est bornée sur [0, 1],

comme le montrent ses variations : |x. ln x| ≤ e1 .

2ème méthode : on peut recourir au théorème d’intégration ∑∫ ]1,0]nu < +∞, qui s’applique puisque la

série ∑+∞

=1

1n

nn converge.

3) Calcul numérique très facile via le critère des séries alternées. Inutile de dire que la série converge vers l’intégrale bien plus vite que les sommes de Riemann. Historiquement d’ailleurs, au XVIIème siècle, on calculait souvent les intégrales par dévelop-pement en série de la fonction, avant de disposer de théorèmes rigoureux.

> J:=int(x^x,x=0..1);evalf(J); plot([x^x,0],x=0..1,thickness=2);

.7834305107 > S:=n->sum((-1)^(k-1)/k^k,k=1..n); for n from 1 to 10 do evalf(S(n));od;

Exercice 2 : Montrer que : ∫+∞

−0.

1dx

exx

= ∑+∞

=1 ²1

n n =

6²π .

Solution : 1) Formellement

∫+∞

−0.

1dx

exx

= ∫∞+

−0.

1.

dxeex

x

x

= ..0∫+∞

−xex ∑+∞

=

0n

nxe .dx = ∫+∞

0 ∑+∞

=

1

.n

nxex .dx

= ∑+∞

=1n

dxex nx..0∫+∞

− = ∑+∞

=1 ²1

n n , après un bref calcul.

dxex nx..0∫+∞

− = ²

1n

dueu u..0∫+∞

− = ²

1n

( chgt de var u = nx, puis intégration par parties ).

2) Or un(x) = x.e−nx est une fonction continue et intégrable sur R+ .

La série ∑+∞

=1

)(n

n xu converge simplement sur R*+ , et a pour somme la fonction f(x) = x

x

eex

−1.

.

Enfin : ∑∫+∞

=1

.)(n I

n dxxu = ∑∫+∞

=

+∞−

1 0.

n

nx dxxe = ∑+∞

=1 ²1

n n < +∞.

Le théorème d’intégration terme à terme des séries s’applique : f est intégrable et...

NB : 1) Par continuité f(0) = 1, tandis que ∑+∞

=1

)0(n

nu = 0. D’où la nécessité de se placer sur R*+ .

2) On peut éviter tout recours à un théorème, en raisonnant élémentairement ainsi :

f(x) est continue en 0, et intégrable sur ]0, +∞[ car 0 < f(x) < exp(−x/2) pour x assez grand.

L’identité exacte : (∀x > 0) f(x) = ∑=

−n

k

kxex1

. + 1

.−−

x

nx

eex

s’intègre terme à terme par linéarité :

∫+∞

−0.

1dx

exx

= ∑∫=

+∞−

n

k

kx dxxe1 0

. + ∫∞+ −

−0.

1.

dxeexx

nx

= ∑=

n

k k1 ²1 + dxexf nx.).(

0∫+∞

− .

1. .7500000000 .7870370370 .7831307870 .7834507870

.7834293536 .7834305678 .7834305082 .7834305108 .7834305107

Page 48: Exercices intégrales généralisées

48

Or 0 ≤ dxexf nx.).(0∫+∞

− ≤ || f ||∞ dxe nx.0∫+∞

− = n1 || f ||∞ → 0 . Conclure en faisant tendre n vers +∞.

Exercice 3 : Montrer que ∫∞+

−0.

1dx

exx

α = Γ(α + 1).ζ(α + 1) pour α > 0.

Solution : Cet exercice généralise le précédent. La solution est identique. A noter que la formule obtenue relie les fonctions ζ et Γ. D’autres liens entre ces deux fonctions permettent de prolonger la fonction ζ. Les formules pour α = 3 et 5 servent en physique pour le rayonnement du corps noir (Boltzmann, loi de Stefan, cf. CCP 2007). Ah ! que j’aime voir rayonner les corps dans le noir…

Exercice 4 : Soit un = )!2(

!...!2!1+

+++n

n . Montrer que la série ∑+∞

=1n

nu converge, et que :

e − 2 + ∑+∞

=1n

nu = − ∫(0,1) et.ln t.dt .

Solution :

Exercice 5 : Montrer ∫(0,1) 1²ln².−x

xx .dx = ∑+∞

= +1 )²12(1

n n =

8²π − 1.

∫(0,1) ln(x).ln(1 − x).dx = ∑+∞

= +1 )²1(1

n nn = 2 −

6²π .

∫(0,1) xxx )1ln(.ln −

.dx = ∑+∞

=13

1n n

(constante d’Apéry ζ(3)).

∫(0,1) ²²)1ln(²).ln(

xxx −

.dx = ∑+∞

= −1 )²12(2

n nn .

∫(0,1) − ²1ln

xx

+ .dx = ∫(0,1) xxArctan .dx = ∑

+∞

= +−

0 )²12()1(

n

n

n (constante de Catalan)

Solution :

1) Formellement, écrivons ∫(0,1) 1²ln².−x

xx .dx = − ∫(0,1) ²1ln².x

xx− .dx = − ∫(0,1) ∑

+∞

=1

2lnn

nxx .dx

= − ∑∫+∞

=1 )1,0(

.2 .lnn

n dxxx = ∑+∞

= +1 )²12(1

n n =

8²π − 1.

La dernière égalité découle de 6²π = ∑

+∞

=1 ²1

n n = ∑

+∞

=1 )²2(1

n n + ∑

+∞

= +0 )²12(1

n n =

41

6²π + ∑

+∞

= +0 )²12(1

n n.

L’avant dernière égalité découle de ∫ )1,0(

2 .ln dxxx n = )²12(

1+n

, qui se montre par parties.

Le nœud de l’affaire tient dans l’application du théorème d’intégration terme à terme des séries à

∑+∞

=1

)(n

n xu , où un(x) = x2n

.ln x sur I = ]0, 1[. Chacune de ces fonctions est intégrable sur I, car

prolongeable par continuité en 0 et en 1, ∑+∞

=1

)(n

n xu = ²1

ln².x

xx− , et enfin

∑∫+∞

=1

.)(n

nI

dxxu = ∑+∞

= +1 )²12(1

n n < +∞.

Page 49: Exercices intégrales généralisées

49

Autre approche : le chgt de variable t = − ln x donne ∫(0,1) 1²ln².−x

xx .dx = ∫+∞

−0 2

3

.1

dte

tet

t

Il reste à développer la fonction en série géométrique, comme dans l’exercice 2. 3) Formellement,

∫(0,1) xxx )1ln(.ln −

.dx = ∫(0,1) xxln ∑

+∞

=

−−1

1)1(

n

nn

nx

.dx = ∑+∞

=

−−1

1)1(

n

n

n ∫ −)1,0(

1 .ln dxxxn = ∑+∞

=13

1n n

.

Le théorème d’intégration terme à terme des séries s’applique sans difficulté. Exercice 6 : Calculer, c’est-à-dire développer en série :

∫(0,+∞) −ln(th x).dx , ∫(0,+∞) shxx

.2.dx

F(x) = ∫(0,+∞) chtxtch )(

.dt , G(x) = ∫(0,+∞) shtxtsh )(

.dt ,

A = ∫(0,1) ²1ln

xx

− .dx , B = ∫(0,1) xx

−1ln .dx , C = ∫(0,1) x

x+1

ln .dx .

Solution : Exercice 7 : Constantes de Nielsen-Ramanujan.

Montrer que les intégrales ak = ∫ −2

1.

1ln

dxx

xk

( k ≥ 1 ) convergent, et que a1 = ζ(2)/2 , a2 = ζ(3)/4.

Solution : Remarque : Pour k ≥ 3, les formules sont moins simples, et font intervenir ζ et les polylogarithmes. (cf. Internet, Mathsoft)

Exercice 8 : Montrer l’identité : (∀x ∈ R) ∫+∞

−0.

1)sin( dt

ext

t = ∑+∞

= +1 ²²n nxx .

Solution : 1) Formellement,

∫+∞

−0.

1)sin( dt

ext

t = ∫∞+

−0.

1).sin(

dte

extt

t

= ∫ ∑+∞ +∞

=

−0 1

.).sin( dtextn

nt = ∑∫+∞

=

+∞−

1 0.).sin(

n

nt dtext = ∑+∞

= +1 ²²n nxx .

En effet, ∫+∞

−0

.).sin( dtext nt = Im ∫+∞ −0

).( .dte tnix = Im nix

tnix−

− ])exp[( +∞

0 = Im ixn−

1 = ²² xn

x+ .

2) Justifcation : Appliquons le théorème d’intégration terme à terme des séries à ∑+∞

=1

)(n

n xu , où

un(x) = sin(xt).exp(−nt). Chacune de ces fonctions est continue et intégrable sur ]0, +∞[.

De plus, ∑∫+∞

=

+∞

1 0.)(

n

n dttu < +∞. En effet, | sin u | ≤ | u |, donc ∫+∞

0.)( dttun ≤ ∫

+∞−

0).exp(.. dtnttx =

²nx

.

Exercice 9 : On admet ∫R exp(−t2).dt = π . Calculer les intégrales ∫R+ t2n.exp(−t2).dt . En déduire que, pour tout z ∈ C, la fonction t → cos(z.t).exp(−t2) est intégrable sur R+ et calculer son intégrale.

Solution :

Exercice 10 : Soit F(a) = ∫(0,1) ln(x).ln(1 − xa).dx , a > 0. Existence de l’intégrale ?

Page 50: Exercices intégrales généralisées

50

Développer F(a) en série. Limite de F(a) en 0 ? Limite et équivalent de F(a) en +∞ ?

Solution :

Exercice 11 : Montrer l’identité : ∫ +1

0 1 αtdt = ∑

+∞

= +−

0 1)1(

n

n

nα pour tout α > 0 .

Solution : Nous sommes ici sur le segment [0, 1].

Formellement : ∫ +1

01 αtdt = ∫ ∑

+∞

=−

1

0 0

)1(n

n tnα dt = ∑ ∫+∞

=−

0

1

0)1(

n

n tnα dt = ∑+∞

= +−

0 1)1(

n

n

nα .

Mais aucun des deux théorèmes d’intégration terme à terme des séries (celui sur les segments, celui sur les intégrales impropres) ne s’applique :

• le reste Rn(t) = (−1)n+1α

α

tt n

+

+

1

)1(

, ne tend pas uniformément vers 0, car sup[0,1] | Rn(t) | = 21 .

• par ailleurs ∑n≥0 ∫I |un(x)|.dx = ∑+∞

= +0 11

n nα = +∞ .

Il reste à montrer les choses élémentairement, i.e. :

∫ +1

01 αtdt = ∫

1

01[ − tα + ... + (−1)n.tnα + (−1)n+1

α

α

tt n

+

+

1

)1(

].dt = ∑= +

−n

k

k

k0 1)1(

α + (−1)n+1∫ +

+1

0

)1(

.1

dtt

t n

α

α

.

Or 0 ≤ ∫ +

+1

0

)1(

.1

dtt

t n

α

α

≤ dtt n .1

0

)1(∫ + α = 1)1(

1++ αn

. cqfd.

On peut aussi procéder par encadrement et gendarmes, 1

1+αt

étant compris entre deux sommes

partielles consécutives de 1 − tα + ... + (−1)n.tnα .

Exercice 12 : Soit (λn) une suite croissante de réels > 0, tendant vers +∞.

Montrer que : ∫ ∑∞+ +∞

=

−−0

0

. ..)1( dxen

xn nλ = ∑+∞

=

−0

)1(

n n

n

λ.

En déduire les sommes ∑+∞

= +−

0 1)1(

n

n

n et ∑

+∞

= +−

0 12)1(

n

n

n. Retrouver l’exercice précédent.

Solution :

0) Le théorème d’intégration terme à termes des séries s’applique si la série ∑+∞

=0

1n nλ converge.

Dans le cas général, il ne s’applique pas. En attendant qu’on adjoigne aux prochains programmes un nouveau théorème qui évitera de réfléchir, il faut se débrouiller tout seul, en se souvenant de cette lettre de Tourgueniev à Tolstoï (1856) :

« Ceux qui s’attachent à des systèmes sont ceux qui, incapables d’embrasser la vérité toute entière, tentent de l’attraper par la queue. Un système, c’est un peu la queue de la vérité, mais la vérité est comme le lézard : elle vous laisse sa queue entre les doigts, et file, sachant parfaitement qu’il lui en poussera une nouvelle en un rien de temps. »

1) Montrons d’abord l’existence des deux membres.

• La série ∑+∞

=

−0

)1(

n n

n

λ obéit au critère des séries alternées.

Page 51: Exercices intégrales généralisées

51

• La série de fonctions ∑+∞

=

−−0

.)1(n

xn ne λ obéit aussi au critère des séries alternées, pour chaque x > 0.

Sa somme f(x) est continue, car la série est uniformément convergente sur toute demi-droite [a, +∞[,

a > 0, car le reste Rn(x) = ∑+∞

+=

−−1

.)1(nk

xk ke λ tend uniformément vers 0 : | Rn(x) | ≤ xne 1+−λ ≤ ane 1+−λ .

De plus, 0 ≤ f(x) ≤ xe 0λ− , donc f est intégrable.

2) Reste à montrer l’égalité des deux membres. Et là, il suffit d’utiliser les gendarmes !

Pour tout x et tout N : ∑+

=

−−12

0

.)1(N

n

xn ne λ ≤ f(x) ≤ ∑=

−−N

n

xn ne2

0

.)1( λ .

Il reste à intégrer l’inégalité et à passer à la limite. 3) Applications :

∑+∞

= +−

0 1)1(

n

n

n = ∫ ∑

+∞ +∞

=

+−−0 0

).1( ..)1( dxen

xnn = ∫+∞

+0.

1dx

ee

x

x

= ∫ +1

01 tdt = ln 2.

∑+∞

= +−

0 12)1(

n

n

n = ∫ ∑

+∞ +∞

=

+−−0 0

).12( ..)1( dxen

xnn = ∫+∞

+0 2 .1

dxee

x

x

= ∫ +1

0 21 tdt =

4π .

∑+∞

= +−

0 1)1(

n

n

nα = ∫ ∑+∞ +∞

=

+−−0 0

).1( ..)1( dxen

xnn α = ∫+∞

+0.

1dx

ee

x

x

α = ∫ +1

01 αtdt .

Exercice 13 : Pour tout n ≥ 1 et tout x ∈ R on pose un(x) = e−nx − 2.e−2nx .

1) Montrer que ∑+∞

=1

)(n

n xu converge pour tout réel x > 0, et calculer sa somme f(x) .

2) Montrer que chaque un est intégrable sur R+, ainsi que f .

3) Comparer ∫+∞

0).( dxxf et ∑

+∞

=1n∫

+∞

0).( dxxun . Expliquer ce résultat .

Solution : Les exercices précédants montrent que l’on peut parfois intégrer terme à terme une série de fonctions sans que les théorèmes du cours ne s’appliquent. L’exercice qu’on va traiter fournit un

exemple très simple dans lequel ∫ ∑+∞ +∞

=0 1

).( dxxun

n ≠ ∑+∞

=1n∫

+∞

0).( dxxun .

1) Convergence simple. Pour chaque x > 0, ∑+∞

=1

)(n

n xu converge comme combinaison linéaire de

séries géométriques convergentes, et ∑+∞

=1

)(n

n xu = x

x

ee

−1 − 2 x

x

ee

2

2

1 −

− = 1

1+xe

.

On prendra garde toutefois que cette formule n’est pas vraie en 0 : un(0) = −1, tandis que f(0) = 21 .

2) Intégrabilités évidentes : un est combinaison linéaire de fonctions intégrables.

3) On a ∫+∞

0).( dxxun =

n1 − 2

n21 = 0. Donc ∑

+∞

=1n∫

+∞

0).( dxxun = 0.

Tandis que ∫ ∑+∞ +∞

=0 1

).( dxxun

n = ∫+∞

0 1+xedx = ∫

+∞

1 )1( +ttdt = ln 2 ( chgt de var. t = e

x ). Et voilà !

Explication : ∫+∞

0.)( dxxun =

n1 ∫

+−

1

0.21 dss =

n21 , donc ∑ ∫

+∞

0.)( dxxun = +∞.

Il est curieux de constater que Maple s’y laisse prendre un instant…

> f:=x->exp(-x)/(1-exp(-x))-2*exp(-2*x)/(1-exp(-2*x));

Page 52: Exercices intégrales généralisées

52

:= f → x − eeee

( )−x

− 1 eeee( )−x

2 eeee( )−2 x

− 1 eeee( )−2 x

> int(f(x),x=0..infinity);ff:=simplify(f(x));int(ff,x=0..infinity); 0

:= ffeeee

( )−x

+ eeee( )−x

1

( )ln 2 > with(plots):u:=(k,x)->exp(-k*x)-2*exp(-2*k*x): S:=(n,x)->sum(u(k,x),k=1..n):p:=n->plot(S(n,x),x=0..3,color=blue): q:=plot(ff,x=0..3,color=red,thickness=2):display([q,seq(p(n),n=1..6)]);

9. Intégrales impropres à paramètres.

Exercice 1 : Étudier la fonction F(x) = ∫+∞

∞− ++ dtxtt

t .²)²)(1²(

² .

Solution : [ ENSAE 1991 ] F(x) est l’intégrale d’une fraction rationnelle. Elle se calcule élémentairement ! Point n’est besoin de se ruer sur les théorèmes du programme, dans lesquels s’embourba jadis un candidat maladroit ! Maple fournit les éléments de preuve : > F:=x->int(t^2/((t^2+1)*(t^2+x^2)),t=-infinity..infinity);F(1);F(x);

12

π

−π

− x 1 < x 0

2 π = x 0

π + x 1

< 0 x

> convert(t^2/((t^2+1)*(t^2+x^2)),parfrac,t);

− + 1

( ) − x 1 ( ) + x 1 ( ) + t2 1

x2

( ) − x 1 ( ) + x 1 ( ) + t2 x2

Exercice 2 : Soit f une fonction R → R réglée et intégrable.

Montrer que sa transformée de Fourier F(x) = ∫+∞

∞−− dtetf ixt.).( est définie, continue et bornée sur R.

Page 53: Exercices intégrales généralisées

53

Montrer que, si f est de plus à décroissance rapide, i.e. si, pour tout entier p, f(t) = O(1/tp) en ±∞,

alors F est de classe C∞

.

Solution : Exercice 3 : Intégrale de Frullani.

Convergence et calcul de ∫∞+ −− −

0.dt

tee btat

pour a et b > 0.

On pourra faire varier a, ou considérer F(x) = ∫+∞

−−− −

0. dte

tee

xtbtat

.

Solution : Nous avons déjà rencontré cette intégrale au § 4.

1) Fixons b et considérons ∫∞+ −− −

0.dt

tee btat

comme une fonction I(a) de a, alors

I’( a) = ∫+∞

−−0

.dte at = −a1 , donc I(a) = − ln a + cte. Comme I(b) = 0, I(a) = ln b – ln a.

Reste à montrer que le théorème de dérivation des IAP s’applique.

Considérons la fonction f(a, t) = tee btat −− −

; on a ),( taaf

∂∂

= − ate− .

Les hypothèses H1, H2 et H3 s’appliquent à ces deux fonctions. Vérifions H3 :

Si K est un compact de R*+, K ⊂ [α, Α], alors, pour tout (a, t) ∈ K×R*+ ,

tee btAt −− −

≤ f(a, t) ≤ tee btt −− −α

, donc | f(a, t) | ≤ max (tee btt −− −α

, −tee btAt −− −

)

Et | ),( taaf

∂∂ | = ate− ≤ te α− . Voilà nos deux majorantes intégrables !

2) La fonction F(x) = ∫+∞

−−− −

0. dte

tee

xtbtat

est définie sur R+. Dérivons et réintégrons !

F’(x) = ∫+∞ +−+− −0

)()( ).( dtee txatxb = xb+

1 − xa+

1 , donc F(x) = ln(x + b) – ln(x + a). Il reste à faire x = 0.

Ici aussi, la dérivation sous l’intégrale reste à justifier.

Exercice 4 : Montrer que F(x) = ∫∞+

∞−

+dt

tx

e t

est définie et continue sur ]0, +∞[.

Solution :

Exercice 5 : Etudier la fonction F(x) = ∫∞+ −

+0.dt

xte t

. Domaine, propriétés, variations, graphe.

Montrer qu’au V(0+), F(x) = − ln x + ∫+∞

−0

.ln. dtte t + o(1).

Solution :

Exercice 6 : Montrer que la fonction F(x) = ∫+∞

+−

0.

)()(

dtxtttEt

est C∞

sur R*+.

Limites et équivalents en +∞ et en 0+ ?

Solution : [ Oral 2005, RMS n° 479 ]

Page 54: Exercices intégrales généralisées

54

Exercice 7 : On considère la fonction : F(x) = ∫+∞

−0

².).cos( dtext t .

Domaine de définition ? Montrer que F est de classe C1 et vérifie une équation différentielle.

En déduire une expression de F(x) ( On admet que ∫R e−t².dt = π ).

Solution :

Exercice 8 : On considère la fonction : F(x) = ∫+∞

−0

².).( dtextch t .

Domaine de définition ? Etablir que : F(x) =2π ex²/4.

Solution :

Exercice 9 : Montrer l’identité de Legendre : ∫+∞

−0

2/² .).sin( dtext t = e−x²/2∫x

t dte0

2/² . (∀x ∈ R)

Solution : La fonction F(x) = ∫+∞

−0

2/² .).sin( dtext t est définie sur R.

En effet, pour tout x, la fonction f(x, .) : t → sin(xt). 2/²te− est continue et intégrable : | f(x, t) | ≤ 2/²te− .

De plus xf

∂∂

(x, t) = − t.sin(xt). 2/²te− est également intégrable. Les hypothèses (H1), (H) et (H3) sont

vérifiées par f et xf

∂∂

, donc F est C1 et F’(x) = ∫

+∞−−

0

2/² .).cos( dtextt t .

Une I.P.P. donne F’(x) = x.F(x) – 1, F(0) = 0. Si l’on intègre cette équation différentielle par les méthodes classiques, on trouve le second membre. Exercice 10 : Transformée de Fourier d’une gaussienne.

Soit a > 0. Convergence et calcul de F(x) = ∫+∞

∞−−− dtee ixtat .² .

Solution : La fonction ²ate− est intégrable et à décroissance rapide.

En vertu de ce qui précède, F(x) = ∫+∞

∞−−− dtee atixt .² est définie, C

∞ sur R, et tend vers 0 en ±∞.

Reste à calculer F(x). 1ère méthode : équation différentielle.

F’(x) = ∫+∞

∞−−−− dteite atixt .² = ( IPP ) = −

ax

2F(x) .

C’est une équation différentielle linéaire homogène d’ordre 1.

Comme F(0) = ∫+∞

∞−− dte at .² =

aπ , F(x) = F(0). a

x

e 4²− =

aπ a

x

e 4²−.

2ème méthode : développement en série. Formellement :

∫+∞

∞−−− dtee atixt .² = ∫ ∑

∞+

∞−−

+∞

=

−dte

nixt

at

n

n

.!)(

²

0

= ∑+∞

=

−0 !

)(

n

n

nix

∫+∞

∞−− dtet atn .²

= ∑+∞

=

−0

2

)!2()(

p

p

pix

∫+∞

∞−− dtet atp .²2 ( si n est impair, ∫

+∞

∞−− dtet atn .² = 0 )

Page 55: Exercices intégrales généralisées

55

= ∑+∞

=

−0

2

)!2()1(

p

pp

px

2∫+∞

−0

²2 .dtet atp = … = aπ ∑

+∞

=

−0

2

)4(!)1(

pp

pp

apx

= aπ a

x

e 4²−.

car les intégrales ∫+∞

−0

²2 .dtet atp se ramènent à ∫+∞

−0

².dte at par des IPP.

Il reste à justifier l’intégration terme à terme des séries au moyen du théorème ad hoc. 3ème méthode : intégration complexe.

F(x) = ∫+∞

∞−−− dtee atixt .² = ∫

+∞

∞−

−+−dte a

xaixta

.4²)²(

= ax

e 4²−

∫+∞

∞−

+−dte a

ixta.

)²( par mise sous forme canonique. Le

changement de variable u = t + aix donne alors : F(x) = a

x

e 4²−

∫+∞

∞−− due au.² =

aπ a

x

e 4²−.

Cette méthode est hélas erronée, car la variable d’intégration n’est pas réelle ! On peut cependant la rendre rigoureuse, mais il faut passer par l’intégration complexe, en introduisant une intégrale curviligne convenable…

Exercice 11 : On considère la fonction F(x) = ∫∞+ −

+0

²

.²1

dtt

e xt

.

1) Domaine de définition de F ?

2) Montrer que F est continue sur R+, de classe C1 sur R*+, et vérifie une équation différentielle.

3) En déduire la valeur de l’intégrale de Gauss I = ∫R e−t² dt .

Solution : 1) La fonction fx : t → ²1

²

te xt

+−

est continue positive sur R+.

• Si x ≥ 0, elle est intégrable, car 0 ≤ fx(t) ≤ ²1

1t+ intégrable.

• Si x < 0, elle tend vers +∞ en +∞, donc n’est pas intégrable.

Conclusion : F est définie sur R+.

2) La fonction f : (x, t) → ²1

²

te xt

+−

obéit aux hypothèses

(H 1) pour tout x ≥ 0, f(x, . ) est intégrable ; (H 2) pour tout t ≥ 0, f(. , t) est continue ;

(H 3) majorante intégrable : ∀(x, t) 0 ≤ f(x, t) ≤ ²1

1t+ .

Par conséquent, F est continue sur R+ .

La fonction xf

∂∂

: (x, t) → ²1

². ²

tet xt

+− −

obéit aux hypothèses :

(H 1) pour tout x > 0, xf

∂∂

(x, .) est intégrable (car ≤ e−xt

au V(+∞)) ;

(H 2) pour tout t ≥ 0, xf

∂∂

(. , t) est continue ;

(H 3) majorante intégrable ∀a > 0 ∀(x, t) ∈ [a, +∞[×R+ 0 ≤ f(x, t) ≤ ²1

²

te at

+−

.

Par conséquent, F est C1 sur [a, +∞[, donc sur R*+, et F’(x) = − ∫

∞+ −

+0

²

.²1

².dt

tet xt

.

Du coup, F(x) – F’(x) = ∫+∞

−0

².dte xt = xI ( chgt de var t x = u ).

Ajoutons que F(x) → 0 en +∞, soit par convergence dominée (majorante intégrable),

Page 56: Exercices intégrales généralisées

56

soit par les gendarmes : 0 ≤ F(x) ≤ ∫+∞

−0

².dte xt = xI .

Remarque : F est une transformée de Laplace : F(x) = ∫∞+ −

+0.

)1(2ds

ss

e xs

.

3) Intégrons cette équation différentielle. Equation homogène : F(x) = C.e

x.

Variation des constantes : F(x) = C(x).ex donne − C’(x).e

x =

xI , C’(x) = −

xI e

−x , donc

C(x) = I ∫+∞ −

x

t

dtt

e. + A et F(x) = I e

x ∫

+∞ −

x

t

dtt

e. + A e

x

Je dis que ex ∫

+∞ −

x

t

dtt

e. → 0 en +∞, car 0 ≤ e

x ∫

+∞ −

x

t

dtt

e. ≤

x

ex

∫+∞

−x

t dte . = x

1 .

Comme F(x) → 0 en +∞, A = 0 et F(x) = I ex ∫

+∞ −

x

t

dtt

e. .

Mais F est continue en 0 et F(0) = 2π . Donc

2π = I ∫

∞+ −

0.dt

t

e t

= 2 I2 et I =

2π .

Exercice 12 : Intégrale de Poisson : P(x) = ∫(0,π) ln(x2 − 2x.cos θ + 1).dθ .

1) Domaine de définition de P ? Les questions 2 et 3 doivent être traitées indépendamment. 2) a) Montrer que P est continue sur R, dérivable en tout x ≠ ±1 ; calculer P'(x). b) Trouver limx→+∞ P(x) − 2π.ln x . En déduire P(x).

3) a) Montrer que P est paire et vérifie : P(x2) = 2.P(x) et P(1/x) = P(x) − 2π.ln |x| .

b) Retrouver P(x).

Solution : [ Oral Centrale MP 2011, RMS n° 883 ]

Exercice 13 : On considère la fonction : F(x) = ∫ +2/

0).²sin.1ln(

πθθ dx .

1) Domaine de définition de F ? Continuité, dérivabilité ?

2) Montrer que F(x) = π. ln(1 + x+1 ) − ln 2 . Calculer F(−1) .

Solution :

Exercice 14 : Fonction de Wallis : W(x) = ∫2/

0.sin

πdttx .

1) Domaine de définition de W ? Montrer que W est de classe C∞ sur son domaine. 2) Monotonie, convexité, limites ; calcul de W(0), W(1), W'(0) et W'(1) ; graphe ? 3) Relation entre W(x + 2) et W(x) ? Que dire de (x + 1).W(x).W(x + 1) ? 4) Équivalents de W(x) en −1+ et en +∞ ?

Solution :

Remarque : La fonction W est liée à la fonction Bêta : W(x) = 21 B(

21+x ,

21 ).

On peut obtenir un d.a. à tous ordres de W en +∞, par la méthode de Laplace.

Exercice 15 : Montrer les formules : (∀x > 0) ∫+∞

+0.

²1)cos(

dttxt

= ∫+∞

+0.

²1)sin(.

dttxtt

= 2π xe− .

Page 57: Exercices intégrales généralisées

57

Solution :

Exercice 16 : 1) Etudier F(x) = ∫+∞

+0.

²1)tan(

dtt

xtArc: définition, continuité, dérivabilité, limite en +∞.

2) Etudier la fonction G(x) = ∫+∞

+0.

²)1.()tan(

dttt

xtArc. Calculer G’(x).

En déduire G(x), puis la valeur de I = ∫+∞

0(

ttArctan )².dt .

Solution :

Exercice 17 : Domaine de déf. de F(x) = ∫∞+

++

0.

1)1ln(

dtttx

x

. Limites en 1 et en −1 ; équivalents en ±∞.

Solution : [ Oral X 1989, RMS n° 68 ] Cf mes feuilles d’exercices

Exercice 18 : Domaine de définition de F(x) = ∫+∞

+++0 11 xttdt . Continuité.

Equivalent en 0, limite en + ∞.

Solution : [ Oral Centrale 1994, RMS n° 276, Oral X 1998, RMS n° 55 ] Cf mes feuilles d’exercice.

Exercice 19 : Montrer que F(x) = ∫+∞

−0

.²).cos( dtext t est C∞

, mais que sa série de Taylor en 0

diverge.

Solution :

10. Fonctions eulériennes.

Exercice 1 : Stirling par convergence dominée.

1) Par le changement de variable y = x + t x , montrer que Γ(x + 1) = dyey yx ..0

−+∞

∫ s’écrit

(x/e)x. x ∫R dttxf ).,( , où f(x, t) = 0 si t ≤ − x et est à préciser sinon.

2) Déterminer la limite de f(x, t) lorsque x tend vers +∞, t étant fixé.

3) On suppose x ≥ 1. Montrer que

0 < f(x, t) ≤ (1 + t).e−t

si t ≥ 0 , 0 < f(x, t) ≤ e−t²/2

si − x < t ≤ 0.

4) Conclure que Γ(x + 1) ∼ (x/e)x. xπ2 quand x → +∞.

Solution : Il existe de nombreuses preuves de la formule de Stirling. Les plus puissantes passent par la méthode de Laplace. C’est une forme de cette méthode qui est ici proposée. Elle a l’inconvénient d’être un peu dogmatique.

Le changement de variable y = x + t x donne Γ(x + 1) = dyey yx ..0

−+∞

∫ = (ex )x x ∫R dttxf ).,( ,

où f(x, t) = 0 si t ≤ − x , f(x, t) = ( 1 + xt )x

xte− si t ≥ − x .

Page 58: Exercices intégrales généralisées

58

Fixons t. Quand x tend vers +∞, f(x, t) = exp[ x ln (1 + xt ) – t x ] tend vers exp −

2²t .

Supposons x ≥ 1. Je dis que 0 < f(x, t) ≤ ( 1 + t ).e−t

si t ≥ 0 et 0 < f(x, t) ≤ e−t²/2

si − x < t ≤ 0.

A(t) = ln( 1 + t ) − x ln ( 1 + xt ) + t ( x − 1) vérifie A’(t) =

))(1(²).1(xtt

tx++

− ≥ 0.

A est croissante sur R+ ; comme A(0) = 0, A(t) ≥ 0 pour t ≥ 0.

B(u) = ln(1 + u) ≤ u − 2²u pour −1 < u ≤ 0, par étude des variations ou développement en série.

On en déduit B(xt ) ≤ 0 pour − x < t ≤ 0.

Le théorème de convergence dominée s’applique, avec une majorante intégrable

0 < f(x, t) ≤ ϕ(t) , où ϕ(t) = ( 1 + t ).e−t

si t ≥ 0 , e−t²/2

si t ≤ 0.

Par conséquent, ∫R dttxf ).,( → ∫ −R

t dte .2/² quand x → +∞.

Exercice 2 : Formule de Gauss.

1) Montrer que, pour tout α > 0, Γ(α) = lim n→+∞ ∫ −−n

n dttnt

0

1.)1( α .

2) En déduire que Γ(α) = lim n→+∞ ))...(1(

!.n

nn++ ααα

α .

3) Application : calculer Γ(21 ).

Solution :

1) Soit fn(t) = ( 1 − nt )

n t

α−1 pour 0 < t ≤ n , fn(t) = 0 pour t ≥ n.

La suite fn(t) tend simplement vers exp(−t) tα−1

et vérifie (∀t > 0) 0 ≤ fn(t) ≤ exp(−t) tα−1

. Le théorème de convergence dominée conclut.

2) Calculons les intégrales In = ∫ −−n

n dttnt

0

1.)1( α = nα∫ −−

1

0

1.)1( duuu n α .

Une IPP permet de calculer B(α, n+1) = ∫ −−1

0

1.)1( duuu n α . On trouve ))...(1(

!n

n++ ααα .

Attention, si l’on travaille directement ∫ −−n

n dttnt

0

1.)1( α , ça marche mal.

On en déduit que Γ(α) = lim n→+∞ ))...(1(

!.n

nn++ ααα

α .

3) Du coup, Γ(21 ) = lim n→+∞

)12...(5.3.1!..2 2/11

++

nnnn

= lim n→+∞ )!2).(12()²!.(.2 2/112

nnnnn

++

= π via Stirling.

Exercice 3 : Soit a ≥ 0, In = ∫+∞ −

a

xdxe

n

. (n ≥ 1). Equivalent de la suite (In) et nature de la série

∑+∞

=1n

nI . ( Commencer par a = 0, a = 1, 0 < a < 1 et a > 1 ).

Solution : L’intégrabilité de la fonction ne pose aucun problème.

Page 59: Exercices intégrales généralisées

59

Si a = 0, In = ∫+∞ −

0.dxe

nx = )1(.1

nnΓ = )11(

n+Γ ( chgt de variable t = x

n ).

Du coup, par Taylor-Young, In = )1(Γ + n1).1('Γ + … + p

p

np!).1()(Γ + O( 1

1+pn

)

Si a > 0, In est une fonction Γ incomplète ; cf mes notes manuscrites…

Problème 4 : Représentations intégrales de Ψ et Ω.

1) a) Montrer (∀x > 0) Ψ(x) = − γ − x1 + ∫

1

0 tt x

−−11

.dt = − γ − x1 + ∫

1

0 uu x)1(1 −−

.du .

b) En déduire la formule de Stern :

(∀x > 0) Ψ(x) = − γ − x1 + x + ∑

+∞

=2n

(−1)n−1.!.

)1)...(1(nn

nxxx +−−

2) Montrer que : (∀x > 0) Ψ(x) = − γ + ∫+∞

0 )exp(1)exp()exp(

txtt

−−−−−

.dt .

3) Par dérivation sous ∫ , établir la formule de Plana , où Ω = lnΓ :

(∀x > 0) Ω(x) = ∫∞+

0

1

t.[(x − 1).exp(−t) −

)exp(1)exp()exp(

txtt

−−−−− ].dt .

4) Montrer la formule de Cauchy-Binet :

(∀x > 0) Ω(x) = (x − )21 .ln x − x +

21 ln(2π) +

t1

0∫+∞

.(211

11 −−− − te t ).exp(−xt).dt .

[On pourra dériver deux fois les deux membres]. En déduire le développement asymptotique à tous ordres de Ω(x) en +∞.

Solution :

Exercice 5 : Intégrales se ramenant à Γ.

1) Montrer que ∫+∞ −

0

bcte ta−1.dt est définie ssi

ba > 0 et c > 0, et vaut

bacbba/.)/(Γ

.

2) En déduire que, pour tout b > 0, ∫+∞ −

0.dte

bt = Γ(1 +

b1 ) . Cas où b = 2 ?

3) Nature et calcul éventuel de ∫ −1

0

1 .1ln dtt

x et de ∫ −1

0.)ln.( duuu ba .

4) Montrer que : (∀x > 0) Γ(x) = dttextt tx .ln.)..(0

1 −+∞

− −∫ .

5) Montrer que : (∀x > 0) Γ(x) = ∫+∞

∞−− dtext t).exp( .

Solution : Problème 6 : Formule des compléments.

p désigne un réel ∈ ]0, 1[ , et F(x) = dtet

tpxt

p

..1.

0

1

∫∞+

−−

+ .

1) Montrer que F est définie et continue sur R+ , de classe C1 sur R*+ ,

et vérifie : F'(x) − F(x) = − pxp)1( +Γ

(∀x > 0) .

Page 60: Exercices intégrales généralisées

60

2) En déduire : F(x) = Γ(1 + p).ex. ∫+∞

−x

pt dtte .. .

3) En conclure : Γ(p).Γ(1 − p) = ∫∞+ −

+0

1

.1

dtt

t p

.

4) On pose g(p) = ∫ +

−1

0

1

.1

dtt

t p

.

a) Montrer que ∫∞+ −

+0

1

.1

dtt

t p

= g(p) +1

1−p

.g(1−p

p) .

b) Montrer que g(p) = ∑+∞

= +−

01)1(

n

n

np et

11−p

.g(1−p

p) = ∑

+∞

= −+−

0 1)1()1(

n

n

pn .

c) En déduire Γ(p).Γ(1 − p) = 1 + 2.∑+∞

=

−−

1

1

1²²)1(

n

n

pn .

Solution : Problème 7 : Exemples de transformées de Fourier.

1) Soit a un réel > 0. Montrer que, pour tout x ∈ R, la fonction t → ta−1.e−t.eixt est intégrable sur

]0, +∞[. On note Fa(x) = ∫+∞

0ta−1.e−t.eixt .dt .

2) Montrer que Fa est de classe C1, et vérifie l’équation différentielle −( i + x ).F'a(x) = a.Fa(x).

En déduire Fa(x) = Γ(a).( x2 + 1 )a/2.exp(i.a.Arctan x).

3) En déduire les formules suivantes, pour tout x réel :

∫+∞

−0

tet

xt)cos(.dt =

22π .

²11²1

xx

+++ et ∫

+∞−

0

tet

xt)sin(.dt =

22π .

²11²1

xx

+−+ .

Solution :

11. Transformation de Laplace, méthode de Laplace.

Exercice 1 : Soient f ∈ C(R+, R), a réel. On suppose que l’intégrale ∫+∞

−0

.)( dtetf at .

Montrer que, pour tout x > a, l’intégrale ∫+∞

−0

.)( dtetf xt converge.

Solution : [ Oral Navale 2012, RMS n° 951 ]

Si l’intégrale ∫+∞

−0

.)( dtetf at est absolument convergente, la règle de la majorante conclut.

Si elle est semi-convergente, une IPP est nécessaire… Il en résulte que le domaine de définition d’une transformée de Laplace est une demi-droite.

Exercice 2 : Dans cet exercice, x est une variable réelle. On se propose de calculer I = ∫+∞

0.sin dt

tt .

1) a) Domaine de définition de la fonction F(x) = ∫+∞

−0

.sin. dtt

te xt .

b) Montrer que F est de classe C1 sur R*+ ; calculer F'(x).

c) Limite de F en +∞ ? Conséquence ?

2) On note Si(t) = ∫t

duu

u0

.sin pour tout réel t.

Page 61: Exercices intégrales généralisées

61

a) Montrer que G(x) = dttSie xt ).(.0∫+∞

− est définie sur R*+ .

b) Montrer que x.G(x) → I quand x → 0+ . En déduire que F est continue en 0.

c) Applications : calculer I , ∫+∞

−0

.sin. dtt

te t , etc.

Solution : 1) Une transformée de Laplace.

a) Domaine de définition de F(x) = ∫+∞

−0

.sin. dtt

te xt .

La fonction t → t

tsin est continue, et même somme d’une série entière sur R ; elle est bornée car

|t

tsin | ≤ 1. Pour tout x > 0 la fonction t → t

tsin e−xt

est donc intégrable. Pour x = 0, elle est semi-

intégrable. Pour x < 0, elle n’est ni intégrable, ni semi-intégrable, car ∫+

−π

π

)1(.sin.n

n

xt dtt

te ne tend pas

vers 0 quand n → +∞. (poser t = nπ + θ …). Le critère de Cauchy est donc violé.

Conclusion : La fonction F(x) = ∫+∞

−0

.sin. dtt

te xt est définie sur R+.

b) Montrons que F est de classe C1 sur R*+ , et calculons F'(x).

La fonction f : (x, t) → t

tsin e−xt

a une dérivée partielle en x, ),( txxf

∂∂

= − sin t. e−xt

.

Pour tout x > 0 , f(x, .) et ,.)(xxf

∂∂

sont continues par morceaux et intégrables.

Pour tout t > 0 , f( ., t) et )(.,txf

∂∂

sont continues.

Pour tout a > 0 , pour tout x ≥ a et tout t > 0, |f(x, t)| ≤ t

tsin e−at

et | ),( txxf

∂∂ | ≤ sin t. e

−at , qui sont

des majorantes intégrables.

Le théorème de dérivation des intégrales à paramètres s’applique et montre que F est C1 et

F’(x) = − ∫+∞

−0

..sin dtet xt = − Im ∫+∞ −0

).( .dte txi = − Imxi

e txi

− )(

|+∞0 = − Im

ix−1 = −

1²1+x

.

c) Limite de F en +∞ et conséquences.

Elémentairement, |F(x)| ≤ dte xt.0∫+∞

− = x1 , donc F(x) → 0 quand x → +∞.

On peut aussi utiliser le théorème de convergence dominée.

Conclusion : Pour tout x > 0 , ∫+∞

−0

.sin. dtt

te xt = 2π − Arctan x = Arctan

x1 .

On aimerait faire tendre x vers 0 dans cette identité. Si t

tsin était intégrable sur R+, il suffirait de

noter que F(x) est continue sur R+, par convergence dominée. Malheureusement, elle n’est que semi-intégrable, et le programme n’autorise pas ce passage à la limite.

2) Une deuxième transformée de Laplace.

a) Montrons que G(x) = dttSie xt ).(.0∫+∞

− est définie sur R*+ .

La fonction « sinus intégral » Si(t) = ∫t

duu

u0

.sin est continue et même développable en série entière

sur R, et elle a une limite en +∞, donc elle est bornée sur R+.

Page 62: Exercices intégrales généralisées

62

Il en résulte aussitôt que, pour tout x > 0 , t → Si(t) e−xt

est intégrable.

b) Montrons que x.G(x) → I quand x → 0+ .

Un changement de variable donne xG(x) = dttSixe xt ).(.0∫+∞

− = duxuSie u ).(.

0∫+∞

− .

Or e−u

Si(xu ) tend simplement vers I.e

−u quand x tend vers 0+.

Et on a la condition de domination | e−u

Si(xu ) | ≤ || Si ||∞ e

−u .

Donc x.G(x) tend vers duIe u ..0∫+∞

− = I.

Remarque : On peut aussi montrer cela directement par soustraction et cassage en deux.

c) Intégrons par parties !

F(x) = dttSie xt ).('.0∫+∞

− = [e−xt

Si(t)] +∞0 + xG(x) = xG(x) → I = F(0) quand x tend vers 0. cqfd

Conclusion : Pour tout x ≥ 0 , ∫+∞

−0

.sin. dtt

te xt = 2π − Arctan x = Arctan

x1 .

En particulier ∫+∞

0.sin. dt

tt =

2π , ∫

+∞−

0.sin. dt

tte t =

4π , etc.

> with(inttrans); addtable fourier fouriercos fouriersin hankel hilbert invfourier invhilbert, , , , , , , ,[

invlaplace invmellin laplace mellin savetable, , , , ]

> f:=t->sin(t)/t;laplace(f(t),t,x);

:= f → t( )sin tt

> with(plots):S:=x->int(sin(t)/t,t=0..x);S(x); p:=plot(S(x),x=0..40,thickness=2):q:=plot(Pi/2,0..40,color=black): display(p,q);

( )Si x

Exercice 3 : Soit f une fonction [0, +∞[ → C, réglée sur tout segment, vérifiant :

(L) (∃r ∈ R) f(s) = O(ers

) au V(+∞) .

Montrer que sa transformée de Laplace F(x) = ∫+∞

−0

).(. dssfe xs est définie pour x > r, et vérifie :

F(x) = x

f )0( + + o(

x1 ) quand x → +∞ .

Solution : Supposons pour commencer r = 0, autrement dit f bornée.

Alors f(s).exp(−sx) est intégrable pour tout x > 0.

Le changement de variable xs = u donne : F(x) = x1 ∫

+∞−

0).(. du

xufe u .

arctan

1x

Page 63: Exercices intégrales généralisées

63

e−u

.f(xu ) → f(0+).e

−u quand u > 0, e

−u.f(

xu ) → f(0) quand u = 0. De plus | e

−u.f(

xu ) | ≤ || f ||∞.e

−u.

Le théorème de convergence dominée conclut que ∫+∞

−0

).(. duxufe u → ∫

+∞− +

0).0(. dufe u = f(0+).

D’où le résultat.

On peut aussi calculer xF(x) – f(0+) = ∫+∞

− +−0

)).0()(.( dsfsfxe xs , casser en deux et majorer…

Dans le cas général, écrivons F(x) = ∫+∞

−−−0

)( ).(. dssfee rssrx et appliquons ce qui précède à

g(s) = f(s).exp(−rs). Alors F(x) = rx

g−

+)0( + o(

rx−1 ) =

xf )0( +

+ o(x1 ). Cqfd.

Exercice 4 : Limite et équivalent de In = ∫+∞

+0 3)1( ntdt .

Nature et calcul éventuel des séries ∑+∞

=1n

nI , ∑+∞

=

−−1

1)1(n

nn I et ∑+∞

=1n

n

nI .

Solution : [ Oral Mines 2013, RMS n° 582 ]

Nous allons traiter cet important exercice par plusieurs méthodes.

1) Limite.

La fonction fn(t) = nt )1(1

3+ est continue positive et intégrable pour n ≥ 1, car fn(t) ∼ nt31 au V(+∞)

Convergence simple : fn(t) ↓ f(t) , où f(t) = 0 si t > 0 , f(0) = 1.

Domination : 0 ≤ fn(t) ≤ f1(t) pour tout n et tout t.

On conclut que In ↓ 0 en vertu du théorème de convergence dominée. A noter qu’il s’agit de la convergence dominée du pauvre, car il y a convergence uniforme sur tout segment [a, b] ⊂ ]0, +∞[ .

Voici d’ailleurs une preuve directe, élémentaire, de In ↓ 0, par cassage en deux.

In = ∫ε

0+ ∫

+∞

ε≤ ε + 13)1(

1−+ nε ∫

+∞

+ε 31 tdt ≤ ε + 13

1

)1( −+ nIε .

ε > 0 étant fixé, 131

)1( −+ nIε → 0 quand n → +∞, donc 13

1

)1( −+ nIε ≤ ε, et In ≤ 2ε pour n assez grand.

2) Equivalent, 1ère approche. Une intégration par parties permet un calcul récurrent des In.

In = ∫+∞

+0 3)1( ntdt = ∞+

+ 03)1( ntt + 3n∫

+∞

++0 13

3

)1(.

ntdtt

= 3n ( In – In+1 ) .

Finalement 3n In+1 = (3n – 1) In , donc In = I1 ∏−

=−

1

1

)311(

n

k k.

On peut calculer I1, à la main ou avec Maple : I1 = 9

32π .

Passons au log : ln In = ln I1 + ∑−

=−

1

1

)311ln(

n

k k = ln I1 + ∑

=+−

1

1

))²

1(31(

n

k kO

k

= ln I1 − 31 ∑

=

1

1

1n

k k + ∑

=

1

1

1(n

k kO = ln I1 −

31 ln(n − 1) + γ + o(1) + SPSAC = −

31 ln n + A + o(1) .

SPSAC signifie : somme partielle d’une série absolument convergente. Posant K = exp A, il vient :

Conclusion : In = 9

32π ∏

=−

1

1

)311(

n

k k ∼

3 nK , où K est une constante > 0.

Page 64: Exercices intégrales généralisées

64

Remarque : la formule de Gauss )(xΓ = limn→+∞ ))...(1(

!.nxxx

nnx

++ fournit la valeur de K.

On trouve K = 9

32π)3/2(

1Γ .

3) Nature de la série ∑+∞

=1n

nI .

Cette série diverge, en vertu de la règle de l’équivalent. Mais on peut aussi montrer cette divergence directement.

1ère idée : In ≥ ∫+

+1

0 3)1( ntdt ≥ ∫

+−

1

0

3 .)1( dtt n ≥ ∫+

−1

0.)1( dtt n =

11+n

2ème idée : Formellement ∑+∞

=1n

nI = ∑+∞

=1n∫

+∞

+0 3)1( ntdt = ∫ ∑

+∞ +∞

= +0 31 )1( n

n tdt = ∫

+∞

0 3tdt = +∞.

Un argument d’associativité de bornes supérieures valide ce raisonnement.

4) Equivalent par méthode de Laplace.

L’idée fondamentale de cette méthode consiste à noter que In = ∫+∞

+0 3)1( ntdt = ∫

+∞ +−

0

)1ln(.

3

dtetn

.

Or exp(− n ln(1 + t3)) est maximum pour t = 0.

C’est au V(0+) que se concentre la masse quand n tend vers +∞.

On peut donc penser que In ∼ ∫+∞ −

0

..

3

dtetn

= 3/1.3)3/1(

après changement de variable n t3 = s.

On peut justifier cet équivalent de différentes façons.

1ère méthode : découpe à la Chasles. Tout d’abord ∀t ≥ 0, ln( 1 + t

3 ) ≤ t

3 . D’autre part, par définition d’un équivalent en 0 :

∀ε ∈ ]0, 1[ ∃α > 0 ∀t ∈ [0, α] ( 1 − ε) .t3 ≤ ln( 1 + t

3 ).

Donc ∫+∞ −

0

..

3

dtetn

≤ In = ∫+∞ +−

0

)1ln(.

3

dtetn

≤ ∫−−α ε

0

)1(.

3

dtetn

+ ∫+∞

+α ntdt

)1( 3 .

3/1.3)3/1(

≤ In ≤ ∫+∞ −−

0

)1(.

3

dtetn ε

+ 13)1(1

−+ nα ∫+∞

+α 31 tdt ≤ 3/1.3

)3/1(n

Γ3/1)1(

1ε− + 13)1(

1−+ nα ∫

+∞

+0 31 tdt .

Par comparaison exponentielle-puissance, 13)1(1

−+ nα est négligeable devant 3/1.3)3/1(

Donc, pour n assez grand 3/1.3)3/1(

≤ In ≤ 3/1.3)3/1(

nΓ [ 3/1)1(

1ε− + ε] .

Or 3/1)1(1ε− + ε est de la forme 1 + ε’. La preuve est complète.

2ème méthode : changement de variable et convergence dominée.

Le changement de variable s = ln( 1 + t3

) donne t = (es – 1)

1/3 et In = ∫

∞+−

−0 3/2)1(3. ds

ee

e s

sns .

Puis ns = u donne In = ∫∞+

−−0 3/2/

/

)1(3. du

ene

e nu

nuu = 3/13

1n ∫

∞+−

−0 3/2/

/

))1((. du

ene

e nu

nuu .

Or e−u

3/2/

/

))1(( −nu

nu

ene

tend simplement vers e−u

.u−2/3

et est majorée par e−u

.u−2/3

, qui est intégrable,

d’intégrale égale à Γ(1/3). En vertu du théorème de convergence dominée,

∫∞+

−−0 3/2/

/

))1((. du

ene

e nu

nuu → ∫

+∞−−

0

3/2 .. duue u = Γ(31 ) .

Page 65: Exercices intégrales généralisées

65

Conclusion : In ∼ 3/1.3)3/1(

= 3/1

)3/4(n

Γ quand n → +∞.

5) Développement asymptotique à tous ordres de In.

Revenons à l’expression : In = ∫+∞ −−

−0 3/2)1(

)1(31. ds

ee s

sn .

Elle fait apparaître In comme la transformée de Laplace évaluée en n − 1 de f(s) = 3/2)1(31−se

.

Or f(s) admet un développement asymptotique en 0+ de la forme :

f(s) = 3/20

sa + a1 s

1/3 + a2 s

4/3 + a3 s

7/3 + … + ap s

p−2/3 + O(s

p+1/3) .

En réalité, on peut écrire : f(s) = 3/20

sa + a1 s

1/3 + a2 s

4/3 + a3 s

7/3 + … + ap s

p−2/3 + g(s) ,

où g(s) est O(sp+1/3

) au V(0+) et aussi au V(+∞), car g(s) → 0 en +∞.

Par suite, il existe une constante B > 0 telle que, pour tout s > 0, | g(s) | ≤ B.sp+1/3

.

Et ∫+∞ −−0

)1( )(. dssge sn ≤ B ∫+∞ +−−0

3/1)1( .. dsse psn = B. 3/4)1()3/4(

+−+Γ

pnp

= O( 3/41+pn

).

Par linéarité de la transformée de Laplace :

In = a0 3/1)1()3/1(

−Γn

+ a1 3/4)1()3/4(

−Γn

+ a2 3/7)1()3/7(

−Γn

+ a3 s

7/3 + … + ap 3/1)1(

)3/1(+−

+Γpn

p + O( 3/4

1+pn

).

Exercice : Convergence et calcul de ∑+∞

=

−−1

1)1(n

nn I , ∑+∞

=1

.n

nn xI et ∑+∞

=1n

n

nI .

6) Avec Maple : > with(plots):f:=(n,t)->1/(1+t^3)^n; p:=n->plot(f(n,t),t=0..5,thickness=2):display([seq(p(n),n=1..6)]);

> int(f(1,t),t=0..infinity);

29

π 3

> J:=n->2*Pi*sqrt(3)/9*product(1-1/(3*k),k=1..n-1); simplify(J(n));simplify(asympt(J(n),n),exp);

29

π 3

Γ − n

13

( )Γ n Γ

23

:= J → n29

π 3

= k 1

− n 1

− 1

13

1k

29

π 3

1n

( )/13

Γ

23

481

π 3

1n

( )/43

Γ

23

14729

π 3

1n

( )/73

Γ

23

14019683

π 3

1n

( )/103

Γ

23

+ + +

364177147

π 3

1n

( )/133

Γ

23

O

1n

( )/16 3

+ +

Page 66: Exercices intégrales généralisées

66

> with(inttrans);f:=s->1/3*exp(s)/(exp(s)-1)^(2/3): DL:=series(f(s),s=0,9);laplace(DL,s,n);

addtable fourier fouriercos fouriersin hankel hilbert invfourier invhilbert, , , , , , , ,[

invlaplace invmellin laplace mellin savetable, , , , ]

DL13

1

s( )/2 3

29

s( )/1 3 7

108s

( )/4 3 5486

s( )/7 3 13

14580s

( )/10 3 121870

s( )/13 3

+ + + + + :=

24744089920

s( )/16 3 11

13226976s

( )/19 3( )O s

( )/22 3 + + +

29

π 3

n( )/1 3

Γ

23

481

π 3

n( )/4 3

Γ

23

14729

π 3

n( )/7 3

Γ

23

14019683

π 3

n( )/10 3

Γ

23

364177147

π 3

n( )/13 3

Γ

23

7281594323

π 3

n( )/16 3

Γ

23

+ + + + +

1284443046721

π 3

n( )/19 3

Γ

23

108680387420489

π 3

n( )/22 3

Γ

23

( )laplace , ,( )O s( )/22 3

s n + + +

___________

Quelques identités tombées du ciel …

(∀x ≥ 0) ∫∞+ −

+0.

²1ds

se xs

= ∫+∞ −x

dtt

xt.

)sin( = du

xuu.sin

0∫+∞

+

(∀x ≥ 0) ∫+∞

−0

..sin dtet

t xt = 2π − Arctan x.

(∀x > 0) ∫+∞

−−0

..cos1 dtet

t xt = − ln x + 21 ln( x2 + 1 ).

(∀x ≥ 0) ∫+∞

−−0

..²

cos1 dtet

t xt = 2π − Arctan x + x.ln x − ²)1ln(.

2xx + .

(∀x ∈ R) ∫+∞

+0.

²1)cos(

dstxt

= ∫2/

0).tan.cos(

πdttx = xe−.

2π .

2π ( 1 − e

−x ) pour x > 0

(∀x ∈ R) ∫+∞

+0.

²)1().sin(

dttttx

= 0 pour x = 0

2π ( e

x − 1 ) pour x < 0

(∀x > 0) ∫+∞

+0 4 .)1()sin(

dtttxt

= 2π .( 1 − e

−x/ 2.cos

2x )

(∀x ∈ R) ∫+∞

−0

² ).(. dttxche t = 2π .e

x²/4 et ∫

+∞−

0

² ).cos(. dttxe t = 2π .e

−x²/4

(∀x ∈ R) ∫+∞

−0

2/² ).sin(. dttxe t = e−x²/2∫

xt dte

0

2/² . (Legendre)

(∀x ∈ R) ∫2/

0.

sin)sin.tan(π

dtt

txArc = Argshx.

(∀x ≥ 0) ∫+∞

+0.

²1)/tan(

dtt

txArc = ∫ −

xdt

tt

0.

1²ln

Page 67: Exercices intégrales généralisées

67

(∀x > 0) ∫ ++x

dttxt

0.

²1)1ln(

= 21 .ln( 1 + x

2 ).Arctan x .

(∀x ≥ 0) ∫+∞

++

0.

²)1.(²)1ln(

dtttxt

= −21 ∫ −

xdt

tt

0.

1ln .

(∀x > −1) ∫ −1

0.

ln1. dtt

tt x = ln12

++

xx .

(∀a, b > 0) (∀x > max(−a, −b)) ∫∞+

−−− −

0. dte

tee

xtbtat

= lnaxbx

++ .

(∀x ≥ −1) ∫ +2/

0).²sin.1ln(

πdttx = π.ln

211 x++ .

(∀a, b > 1) ∫ −−π

0).

coscosln( dx

xbxa = ).( ArgchbArgcha−π

(∀x ∈ R) ∫+∞

∞− ++

dtt

txArc.

²1)tan(

= π.Arctan2x .

(∀x ∈ R) ∫+∞

∞− + dtt

txArct.

1).tan(.

4 = π.Arctan(x 2 + 1) −

4²π pour x ≥ 0

π.Arctan(x 2 − 1) + 4²π pour x ≤ 0.

(∀x ≥ 0) ∫+∞

+−0

).²(expt

dttxt = xe 2. −π & ∫

+∞+−

0).

²²(exp dt

txt = xe 2. −π

(∀x, a > 0) ∫∞+

+0.

1dt

te

a

xta

= Γ(1 +a1 ) dtet tx

x

a ../1 −+∞

−∫

(∀x > 0) (∀u ∈ R) ∫+∞

∞−−− dtee utixt .. 2² ππ =

x1 xue /²π− .

(∀x ∈ R) ∫∞+ −

0.. dte

t

eitx

t

= 4 ²1 x+

π [cos2tanxArc + i.sin

2tanxArc ].

(∀x > 0) xxln = ∫

+∞

+0.

)²(ln dt

xtt et

2²ln x = ∫

+∞

+−+0.ln).

)1

11( dtt

xtt

_____________